NBDHE Practice Test MEGA-SET 💪🏻

Ace your homework & exams now with Quizwiz!

In a periapical radiograph, increasing the vertical angle when using the bisecting angle technique will cause A. elongation B. overlapping C. foreshortening D. apical shifting of the image E. None of the choices

C. foreshortening

Squamous cell carcinoma can be found in all of these areas EXCEPT A.) Buccal mucosa B.) Floor of the mouth C.) Bone D.) Lateral border of tongue

C.) Bone

Which drug is a calcium channel blocker? A.) Hydrochlorothiazide B.) Propanolol C.) Diltazem D.) Losartan

C.) Diltiazem

Which of the following is NOT a function of a mirror? A.) Soft tissue retraction B.) Indirect vision C.) Direct illumination D.) Transillumination

C.) Direct illumination

Which of the following conditions is characterized by polydactyly and natal teeth? A.) Down syndrome B.) Cyclic neutropenia C.) Ellis-van Creveld syndrome D.) Cherubism

C.) Ellis-van Creveld syndrome

Which of the following agents is usually NOT related to gingival enlargement? A.) Cyclosporine B.) Phenytoin C.) Methylprednisolone D.) Calcium channel blocker

C.) Methylprednisolone

According to the American Heart Association guideline, which of the following heart conditions does NOT require premedication? A.) Incompletely repaired cyanotic congenital heart disease B.) History of infective endocarditis C.) Rheumatic heart disease D.) Artificial heart valve

C.) Rheumatic heart disease

The cathode is composed of all of the following EXCEPT A.) Tungsten filament B.) Molybdenum cup C.) Tungsten target

C.) Tungsten target

Each of the following statements accurately describes aspects of the mineralization of plaque biofilm as it transitions to calculus EXCEPT one. Which one is the EXCEPTION?

Calcium and phosphorus come from the blood for mineralization of subgingival calculus

Gypsum material used in dentistry is called

Calcium sulfate hemihydrate

Direct injury to periodontal tissue is caused by each of the following EXCEPT one. Which one is the EXCEPTION?

Calculus

The radiopaque areas on the mesial surface of tooth #18, and the distal surface of tooth #19 are the result of which of the following? (case J)

Calculus (case J)

When comparing the first photo with the second, what can the dental hygienist conclude about the patient's gingival healing? (case E)

Calculus and plaque biofilm have been removed in the second photo, the healing process can begin (case E)

Calculus removal is important in preventing and treating periodontal disease because

Calculus is a local factor that facilitates the accumulation of plaque

What is the guideline in the administration of nitroglycerine?

Can be given up to 3 tablets in 15 minutes

Each of the following are accurate statements regarding Varenicline (Chantix) prescribed as an aid in smoking cessation therapy?

Can be used in conjunction with other smoking cessation products

Which of the following statements accurately describes gram-positive cell walls?

Cell wall consists of a lipoprotein-lipopolysaccharide-phospholipid outer membrane

The sequence of eruption of the mandibular teeth in the primary dentition is

Central incisor, lateral incisor, first molar, canine, second molar

All of the following agents can be delivered via a cannula/syringe into periodontal pockets EXCEPT one. Which one is this EXCEPTION?

Chlorhexidine chip

Furcation that can only be detected by the probe tip and not penetrate the furcation is

Class 1 furcation

A tooth that is mobile less than 1mm in BL or MD direction is A. Class I B. Class II C. Class III D. None

Class I

Which class of caries occurs on the occlusal surface of posterior teeth, buccal or lingual pits on molars, or the lingual pits of maxillary incisors?

Class I

Using G. V. Black's classification of dental caries and restorations, which of the following describes the restoration on tooth #3? (case A)

Class I (case A)

A sealant is usually placed on permanent molars as soon as they erupt in order to prevent

Class I caries

If the buccal groove of the mandibular first molar is distal to the mesiobuccal cusp of the maxillary first molar, what type of occlusion will the patient have?

Class II

Which class of caries occurs on the interproximal surface of posterior teeth?

Class II

According to G. V. Black's classification of restorations, which of the following describes the restoration on tooth #3? (case D)

Class II (case D)

In assessing the molar relationship of a patient's occlusion you determine that there is a distal relationship of the mesiobucal groove of the mandibular first molar to the mesiobucal cusp of the maxillary first permanent molar. This assessment represents which classification of malocclusion accompanied by which facial profile?

Class II malocclusion with likelihood of a prognathic profile

The classification of dental caries in teeth #29 and #31 can be BEST identified as ________ and ________ respectively as (case C)

Class II, Class II (case C)

The Naber's probe passes through the furcation to the other side, but the furcation is not clinically visible. Which type of furcation does this describe?

Class III

Which class of caries occurs on the interproximal surfaces of anterior teeth?

Class III

During the patient's periodontal evaluation, the hygienist detects a furcation on tooth #30. She notes that while there is through-and-through involvement with complete loss of bone between roots, the opening is covered by gingiva. This type of furcation involvement is classified as

Class III.

Which class of caries occurs on the interproximal surface of anterior teeth including the incisal edge?

Class IV

Which class of caries occurs on the cervical third of facial or lingual surfaces?

Class V

Which class of restorations can glass ionomers be used for?

Class V One of the advantages of glass ionomers is the true bonding between materials and dentin/enamel; thus they have been widely used for Class V restorations which have high requirements in adhesion

Which class of caries occurs on the incisal or occlusal edge due to attrition?

Class VI

Advantages of silver diamine fluoride (SDF) include each of the following EXCEPT one. Which one is the EXCEPTION?

Clear opaque final color is esthetically pleasing

Initially looking at this patient's gingival tissues you suspect his gingival condition may be stage III gingivitis. Which of the following is not consistent with this assessment? (case C)

Clinical signs of gingivitis first appear (case C)

The force of attraction between like kinds of atoms and molecules within the material, resulting in a tenacious bond is called

Cohesion

Which test is MOST widely used in conjunction with electric pulp testing to confirm pulp vitality?

Cold test

The lead diaphragm in the tube head is referred to as the

Collimator

Which of the following does NOT describe likely clinical changes during Stage III gingivitis?Which of the following terms BEST describes inflammatory involvement that includes the attached gingiva, the papilla, and the free gingiva?

Color changes begin in the attached gingiva, then spread to the gingival margin and papilla

All of the following are true regarding the stain present on the lingual surfaces of the mandibular anterior teeth EXCEPT one. Which one is the EXCEPTION (case H)

Coloration is due to blood pigments from the diseased pocket tissues. (case H)

What is the most common scatter produced by dental radiation?

Compton scatter

Which of the following terms describes an oval prominence?

Condyle

What type of breach occurs if the dental hygienist discusses a patient's dental condition with a friend and the identity of the patient is inadvertently revealed?

Confidentiality

Through which of the following routes/mechanisms is tuberculosis not transmitted?

Congenital

On a health history, a patient indicates that his ankles swell, he experiences shortness of breath after climbing a flight of stairs, and that he sleeps with a minimum of two pillows. Which one of the following diseases should be suspected?

Congestive heart failure

Which of the following types of periodontal therapies heals by repair or "scar"?

Connective tissue graft

All of the following are types of subgingival plaque EXCEPT one. Which one is the EXCEPTION?

Connective tissue-attached plaque

Which of the following best describes the relationship between alginate and setting time?

Cooler water temperature increases setting time

A state-hired full-time public health dentist and dental hygienist are responsible for collecting data on the dental plaque rates of elementary school-aged children. Correlational studies are planned to determine strength of data between genders and among geographical locations of the elementary schools. It was decided to screen children in grades K, 2, 4, and 6 within 25 schools throughout the state. The 25 schools were chosen on the basis of geographic locations and number of students enrolled. All students within each of the chosen grades would be part of the study. Three other public health dental hygienists were hired part time to help in the data collection process. When data collection was completed, analyzed, and interpreted, the public health dentist and the dental hygienist would decide on an oral health program to best meet the needs of the elementary school-aged children of the state.

Cross sectional

When measuring the cal on a patient with visible recession, if the probe depth is 7 mm and the recession is 2 mm, what is the cal? A. 2 mm B. 5 mm C. 6 mm D. 9 mm

D. 9 mm

Which of the following materials involves imbibition problems? A. Dental sealant B. Dental composite C. Dental cement D. Alginate impression material E. Gold restorations

D. Alginate impression material

All of the following materials are considered regulated infectious waste except A. Extracted tooth B. Blood-soaked gauze C. Anesthetic needle D. Contaminated exam gloves

D. Contaminated exam gloves

Which one of the following drugs can result in oral candidiasis? A. Cetirizine B. Ibuprofen C. Acetaminophen D. Fluticasone

D. Fluticasone

A ten-year-old girl has an appointment for prophylaxis. this patient reports that for three days she has had severe pain in the attached gingiva surrounding her mandibular anterior teeth. she has an oral temperature of 101°f. the dental hygienist observes that the tissues in the surrounding area bleed easily and are erythematous and ulcerated. which of the following can most likely be expected? A. Aphthous stomatitis B. Juvenile periodontitis C. Early-onset periodontitis D. Herpetic gingivostomatitis E. Necrotizing ulcerative gingivitis

D. Herpetic gingivostomatitis

Powered scaling devices are contraindicated for use in patients with all of the following conditions except A. Myasthenia gravis. B. Cystic fibrosis. C. Emphysema D. Pernicious anemia

D. Pernicious anemia

What phase of the dental hygiene process of care immediately follows diagnosis? A. Assessment B. Evaluation C. Implementation D. Planning

D. Planning

Which of the following is responsible for recording an image on a radiographic film? A. Gelatin B. Adhesive C. Polyester base D. Silver halide crystals

D. Silver halide crystals

The client cannot correctly demonstrate the use of the threader with floss. the discrepancy performance is most likely the result of which of the following? A. Management deficiency B. Motivation C. Readiness to learn D. Skills deficiency

D. Skills deficiency

All of the following are correct in regard to chlorhexidine mouth rinse except one. Which one is the exception? A. Active against gram-positive organisms B. Approved in the U.S. in a concentration of 0.12% C. An antimicrobial antigingivitis agent D. To be used on a long-term basis

D. To be used on a long-term basis

A diabetic client with a glycated hemoglobin assay (HBA1C) test value of 9% means that his/her diabetes is A. Well controlled B. Slightly under control C. Moderately controlled D. Uncontrolled

D. Uncontrolled

Which vitamin is not needed for the development of teeth? A. Vitamin A B. Vitamin D C. Vitamin C D. Vitamin K

D. Vitamin K

Another term for Down syndrome is A. Epicanthus B. disomy 15 C. Autism D. trisomy 21

D. trisomy 21

Which of the following is NOT correct about a right handed clinician that is when working on a patient in a supine position? A.) Ask the patient to tilt the head up for maxillary teeth B.) Ask the patient to tilt the head down for mandibular teeth C.) The occlusal plane is perpendicular to floor D.) 8-9 o'clock is best for anterior surfaces away from the clinician

D.) 8-9 o'clock is best for anterior surfaces away from the clinician

When providing oral care to a patient who has Attention Deficit Hyperactivity Disorder, you should, A.) Schedule short appointments B.) Never leave the patient unattended in the room C.) Use 'tell-show-do' approach D.) All of the above

D.) All of the above

Which inflammatory mediator is related to the destruction of the periodontium? A.) Interleukin-1 B.) Tumor necrosis factor-a C.) Interleukin-8 D.) All of the above

D.) All of the above

An overdose of Pilocarpine can cause A.) Increased salivation B.) Increased lacrimation C.) Increased urination D.) all of the above

D.) All of the above Pilocarpine is a cholinergic drug used in the treatment of xerostomia.

The clinician noticed a cluster of radiolucencies that she called "soap bubble" on the panoramic radiograph of a new patient. Which of the following pathology could be included in the differential diagnosis? A.) Odontoma B.) Leukoplakia C.) Osteosarcoma D.) Ameloblastoma

D.) Ameloblastoma

Which of the following conditions is NOT caused by a virus? A.) Hand-foot-mouth disease B.) Herpes labialis C.) Infectious mononucleosis D.) Candidiasis

D.) Candidiasis

Which type of stroke is caused by a blood clot or debris (located in other parts of the body) which travels through the blood stream and ends up lodging in the narrower brain arteries? A.) Hemorrhagic B.) Thrombotic C.) Ischemic D.) Embolic

D.) Embolic

Which of the following is NOT a calcium channel blocker? A.) Nifedipine B.) Verapamil C.) Diltiazem D.) Enalapril

D.) Enalapril Calcium channel blockers usually end with the suffix -pine Angiotensin converting enzyme (ACE) inhibitors end with the suffix -pril

What symptom is NOT related to excessive fluoride ingestion? A.) Nausea B.) Abdominal pain C.) Diarrhea D.) Fruity breath

D.) Fruity breath

Which of the following instruments is NOT a critical item? A.) Curettes B.) Surgical burs C.) Scalpels D.) Mouth mirrors

D.) Mouth mirrors

Which on of the following is NOT true regarding instrument sharpening techniques? A.) The angle between the instrument face and the stone should be at 110 degrees. B.) The original shape of the instrument should be preserved C.) The internal angle of the cutting blade of a curette should be maintained at 70 degrees. D.) Sharpen from toe to heel of the working end

D.) Sharpen from toe to heel of the working end

Standard precaution provides protection against all of the following EXCEPT A.) Blood B.) Tears C.) Saliva D.) Sweat

D.) Sweat

Which of the following is NOT related to cherubism? A.) It may cause expansion of the mandible B.) It may cause premature loss of primary teeth C.) It is hereditary condition D.) The bone forms radiopacities described as "cotton-wool"

D.) The bone forms radiopacities described as "cotton-wool" Cotton-wool radiopacity is usually referring to the appearance of the bone in Paget disease, not cherubism.

All of the following are true about modified pen grasp EXCEPT A.) Fingers form a soft C-shape B.) Ring finger rests against the middle finger C.) Pad of the middle finger rests on the instrument D.) Thumb and middle finger are place opposite of each other on the handle

D.) Thumb and middle finger are placed opposite of each other on the handle

What modifications is appropriate when treating a hearing-impaired patient? A.) Turn off and take off the hearing aids B.)Do not wear mask C.) Allow to use the sense of touch D.) All of the above

D.) all of the above

Which of the following is the MOST appropriate dental caries index to use with this patient? (case g)

DMFT/deft (case G)

Common signs or symptoms that may occur after tongue piercing include all of the following EXCEPT one. Which one is the EXCEPTION? (case F)

Decreased salivary flow (Case F)

Which one of the following is MOST indicative that an instrument has become dull?

Decreased tactile sensitivity

All of the following conditions are contraindications for dental implant placement EXCEPT one. Which one is the EXCEPTION?

Dental caries

A university dental hygiene school received a state-funded grant to improve the oral health status of senior citizens in a rural area of the United States. The funding was for $50,000 over a 1-year period and covered expenses for salaries, equipment, supplies, and education. It was determined that high rates of root caries, root sensitivity, and gum disease were present in this population. The dental hygiene school decided to address these needs by implementing an oral health program in its clinic and to determine the effectiveness of two different types of desensitizing agents on treating root sensitivity.What is the independent variable in the study on the effectiveness of two different types of desensitizing agents on root sensitivity?

Desensitizing agents

Chronic kidney disease (CKD) can cause clinical manifestations of all of the following conditions EXCEPT one. Which one is the EXCEPTION? (case C)

Diabetes (case C)

Which one of the following statements correctly describes the role of diabetes in the pathogenesis of periodontitis? (case J)

Diabetes can trigger release of pro-inflammatory cytokines (case J)

Which one of the following questions is MOST important to ask a patient with type 1 diabetes? (case J)

Did you take your medication and eat?" (case J)

Atridox is composed of

Doxycycline gel

While performing periodontal debridement of tooth #5, the instrument breaks. All of the following are appropriate actions EXCEPT one. Which one is this EXCEPTION? (case F)

Drying the area using the air-water syringe (case F)

All of the following is correct in regards to chlorhexidine use except one. Which one is the exception? A. Chlorhexidine therapy should be initiated directly after a dental prophylaxis if indicated in treatment plan. B. Patients using Chlorhexidine should be reevaluated and given a thorough prophylaxis at intervals no longer than six months. C. Recommended use is twice daily oral rinsing for 30 seconds D. Usual dose is 15 ml of undiluted Chlorhexidine E. Patients should be instructed to rinse with water or other mouthwashes, brush teeth or eat immediately after using Chlorhexidine

E. Patients should be instructed to rinse with water or other mouthwashes, brush teeth or eat immediately after using Chlorhexidine

The desensitizing agent that depolarizes the cell membrane and transmission of nerve impulse and is most often recommended for hypersensitive teeth is A. Eugenol B. Oxalate C. Silver nitrate D. Sodium fluoride paste E. Potassium nitrate

E. Potassium nitrate

The straight calibrated periodontal probe can be used for all of the following EXCEPT A.) Calculate the width of the attached gingiva B.) Detect calculus C.) Measure the depth of the pocket D.) Measure intraoral lesions E.) Detect caries

E.) Detect caries

Which layer of the trilaminar disk forms the epidermis of the skin?

Ectoderm

During radiographic exposure, if the vertical angulation of the PID is too little or insufficient, the image on the film becomes distorted. Which of the following terms BEST describes the appearance of the tooth?

Elongated

Which layer of the trilaminar disk forms the epithelial linings of the respiratory passages and digestive tract etc.?

Endoderm

Which of the following patient condition factors will present the LEAST difficulty in debriding the mandibular anterior sextant ? (Case H)

Enlarged gingival margins (Case H)

Infectious mononucleosis is caused by

Epstein-Barr virus.

What is the most common location for a dentigerous cyst?

Erupting third molars

Which nutrient CANNOT be synthesized by the body?

Essential amino acids

When evaluating the results of pit and fissure sealant placement, the dental hygienist should do all of the following EXCEPT one. Which one is the EXCEPTION?

Examine the sealant surface to assure a dull and chalky surface exists

At the patient's evaluation following periodontal debridement, gingival bleeding and edema are charted on the mesial and distal surfaces of tooth #13. Which of the following is the recommended treatment? (case F)

Explore for residual calculus and remove any that remains (case F)

A clinician presents a treatment plan that includes the diagnosis, prognosis, fees etc. of the proposed procedure. The client and provider both signed the treatment plan. What type of consent was made in this case?

Express

The mandibular occlusal intraoral photograph of this patient confirms that which of the following treatment needs was performed first? (case C)

Extraction of root tip (case C)

Solar cheilitis is most likely to appear in

Fair-skinned adults

Which type of sterilization method is used only in emergency conditions?

Flash cycle

The abuse of alcohol by a pregnant woman can cause all of the following to the fetus EXCEPT one. Which one is the EXCEPTION?

Flat palate

The dentist has recommended that this patient's parents consult with an orthodontist. After placement of orthodontic appliances, which of the following oral hygiene adjuncts would you NOT recommend? (Case B)

Floss holder (case B)

Short-acting and long- acting β2-adrenergic agonoists (sympathomimetics) prescribed to treat asthma and respiratory conditions include each of the following EXCEPT one. Which one is the EXCEPTION?

Fluticasone (Flovent)

As indicated in the patient's dental history, this patient is a mouthbreather. All of the following are significant oral conditions common in patients who breathe through their mouths EXCEPT one. Which one is the EXCEPTION? (case B)

Fordyce granules (case B)

Criteria for successful treatment outcomes of dental implants include each of the following aspects of the implant-tissue interface EXCEPT one. Which one is the EXCEPTION?

Formation and attachment of the periodontal ligament

All of the following accurately describe aspects of inflammatory papillary hyperplasia of the palate EXCEPT one. Which one is the EXCEPTION?

Frequently becomes malignant with slight pain

Each of the following is true of verruca vulgaris EXCEPT one. Which one is the EXCEPTION?

Frequently occurs intraorally

Each of the following are effects of dehydration EXCEPT one. Which one is the EXCEPTION?

Geographic tongue

What is the side effect of the drug cyclosporine (Sandimmune)?

Gingival hyperplasia

What is the term for soreness of the tongue with inflammation and loss of doral papillae?

Glossitis

Which cell component is responsible for synthesizing lysomes?

Golgi complex

Which one of the following instruments is BEST to remove the subgingival calculus on the mesial surface of tooth #2 (case E)

Gracey 11/12 curet (case E)

A Mission of Mercy (M.O.M.) event was planned for an urban city in a southern state of the United States. This 2-day event was designed to meet the critical needs of underserved people of all ages by providing free dental care to as many adults and children as time, volunteers, and supplies would allow. Local dentists, dental hygienists, dental assistants, dental students, dental hygiene students, and dental assisting students volunteered for the event. Local politicians and two state representatives were also in attendance. Before receiving dental treatment, informed consent was required of all patients. A 10-question patient survey was distributed, and volunteers reviewed each question with patients while the patients were completing the questionnaire. Confidential questions included information on age, gender, education, family income, address, and past dental experiences. At the end of each patient's treatment and before each volunteer completed his or her shift, the patient was asked to complete an evaluation survey. There was an 88% response rate, and 79% answered that their experience was positive.Which of the following information is provided to the participant before signing the informed consent?

Guarantee of confidentiality to each patient

According to the learning ladder model, at what level is behavior modified?

Habit

All of the following are risk factors associated with osteoporosis EXCEPT one. Which one is the EXCEPTION?

Halitosis

Which disease is caused by the Coxsackie virus?

Hand-foot-mouth disease

All of the following are characteristics of free gingiva EXCEPT one. Which one is the EXCEPTION?

Has a stippled surface

Which of the following is an advantage of the porcelain-fused-to-metal crowns observed in this patient compared with porcelain-only crowns?

Has strength to reduce brittleness

This patient wants to use a mouthrinse because he believes it will make his mouth feel more fresh. You have instructed him to use a fluoridated, essential oil-containing mouthrinse without alcohol that carries the American Dental Association (ADA) Seal of Acceptance. Each of the following represents beneficial aspects of your recommendation EXCEPT one. Which one is the EXCEPTION? (case C)

High level of substantivity (case C)

What is the LEAST likely cause of midline shift in this patient?

History of gingival grafting

Which pattern of bone destruction is MOST apparent in this patient's panoramic radiograph? (case D)

Horizontal bone loss (Case D)

Blood pressure of 142/92 mmHg is categorized as

Hypertension stage 2

The administration of oxygen is indicated in most medical emergencies. In which of the following is oxygen NOT indicated?

Hyperventilation

What is myxedema?

Hypothyroidism in adults

What is the ICDAS, and what is the process?

ICDAS: the International Caries Detection and Assessment System is used to classify dental caries. Process 1. Tooth surface cleaned and visually inspected 2. Ball-point probe to examine tooth surface (not explorer!) 3. Score lesion from 0 to 6 4. Recommend treatment based on the score

Which of the following clinical findings indicate failure of a dental implant?

Implant mobility

Which of the following is likely to be the BEST motivator to help this patient improve his home self-care? (case B)

Improvement of appearance (case B)

Some of the radiographic images for this patient appear light (they lack density). What modification in exposure could have prevented this error? (case J)

Increase exposure time ( case J)

Functions of saliva that affect the carious process include all of the following EXCEPT one. Which one is the EXCEPTION?

Increase in acidogenic factors

Adverse reactions to calcium channel blocking agents include all of the following EXCEPT one. Which one is the EXCEPTION?

Increased bleeding

What type of supervision requires that a licensed dentist authorize dental hygiene procedures and remain in the dental facility while those procedures are being performed?

Indirect

Which of the following wound complications should be given first consideration?

Infection

Which of the following BEST describes the gingival marginal tissues on the facial area of teeth #22 through #27? (case G)

Inflamed (case G)

Against the clinician's advice, the patient has declined the procedure because she could not afford it at the moment. This is called

Informed refusal

Bone loss evident on the radiographs on the mesial of tooth #30 is indicative of what type of pocket? (case H)

Infrabony pocket (case H)

In which stage of tooth development do cells condense and proliferate to create the dental lamina?

Initiation stage

Although this patient no longer smokes, all of the following conditions EXCEPT one frequently develop in chronic smokers with COPD. Which one is the EXCEPTION? (case J)

Intrinsic tooth stains (case J)

In a dental radiograph which of the following appears as a radiopaque intersection of the lateral wall of the nasal cavity and the anterior portion of the maxillary sinus?

Inverted Y

Which of the following is related to the dental pellicle?

It helps bacteria attach to the tooth surface. It protects the enamel from acidic activity

The patient presents with an area in the vestibule that is white and tissues that slough off. Upon talking to the patient, you realize that the patient had been suffering from a toothache and he used an aspirin tablet on the area to calm the pain. The patient is worried that he has cancer. What advise could the clinician give?

It is most likely a mucosal burn, therefore the abnormality should be watched for a few weeks as it will heal on its own.

Which primary tooth is in the process of root resorption? (case G)

K (case G)

This patient was diagnosed with stage 2 chronic kidney disease (CKD). What does stage 2 kidney disease imply (case C)

Kidney damage with a mild decrease in kidney function (case C)

Which condition caused by protein deficiency is characterized by edema in the abdomen and legs?

Kwashiorkor

A Mission of Mercy (M.O.M.) event was planned for an urban city in a southern state of the United States. This 2-day event was designed to meet the critical needs of underserved people of all ages by providing free dental care to as many adults and children as time, volunteers, and supplies would allow. Local dentists, dental hygienists, dental assistants, dental students, dental hygiene students, and dental assisting students volunteered for the event. Local politicians and two state representatives were also in attendance. Before receiving dental treatment, informed consent was required of all patients. A 10-question patient survey was distributed, and volunteers reviewed each question with patients while the patients were completing the questionnaire. Confidential questions included information on age, gender, education, family income, address, and past dental experiences. At the end of each patient's treatment and before each volunteer completed his or her shift, the patient was asked to complete an evaluation survey. There was an 88% response rate, and 79% answered that their experience was positive.Dental health in populations such as the one treated by this project is often neglected because of each of the following reasons EXCEPT one. Which one is this EXCEPTION?

Lack of availability of preventive care

A white, painless, flat lesion is best described as,

Leukoplakia

Slim ultrasonic tips are the BEST to remove

Light calculus

A 25-year-old man with a history of gingivitis is assessed at his 6-month recare appointment. Which of the following clinical signs will MOST likely confirm that his condition has progressed to periodontitis?

Loss of clinical attachment

All of the following lesions are palpable during an intraoral exam EXCEPT one. Which one is this EXCEPTION?

Macules

Which permanent teeth occlude with only one tooth in the opposite arch, assuming ideal relations exist?

Mandibular central incisor

Viewing the radiographs, which teeth are expected to exfoliate soon? (case G)

Mandibular primary second molars (case G)

Which of the following is the radiolucent area viewed on the apical area of tooth #20? (case A)

Mental foramen (case A)

Which anatomic feature of the maxillary molar is used to assess the occlusion in Angle's classification?

Mesiobuccal cusp

Which layer of the trilmainar disk forms the muscle, bones, connective tissues, blood vessels etc.?

Mesoderm

Which of the following medications is prescribed for this patient's treatment of high blood pressure (case A)

Metoprolol (case A)

The inability of materials to seal margins adequately around the tooth structure which, in turn, allows leakage of saliva is called

Microleakage

Which of the following is MOST likely the cause of this patient's halitosis? (case E)

Microorganisms that produce foul-smelling compounds (case E)

During which stage does a periapical cemento-osseous dysplasia (cementoma) radiographically appear as a mixed a mixed radiolucent/radiopaque lesion?

Middle stage (osteoblastic stage

Which description of physical status applies to an ASA Class II patient (American Society of Anesthesiologists)?

Mild systemic disease that does not interfere with day-to-day activity; complete a physician consultation before dental hygiene therapy

This patient has Class II mobility on teeth #6 and #7. Which of the following BEST describes this classification of tooth mobility? (case H)

Moderate mobility greater than 1 mm of horizontal displacement (case H)

Each of the following are descriptive of bulimia EXCEPT one. Which one is the EXCEPTION?

Most often found in adolescent females who are overweight

The patient's labial mucosa exhibits an irritation fibroma. Each of the following statements accurately describe this lesion EXCEPT one. Which one is the EXCEPTION? (case F)

Most often ≤ 5 cm in size (case F)

On the facial aspect of tooth #6 a patient's probing depths (PPD) are 4, 4, 4. The gingival margin is 2 mm apical to the CEJ. The mucogingival junction is 3 mm apical to the gingival margin. What is the MOST likely periodontal diagnosis of this area?

Mucogingival involvement

Which of the following is NOT among the three classes of ventilator filters certified by The National Institute for Occupational Safety and Health Certification (NIOSH)?

N-93%

To reduce radiation exposure when taking a full mouth series of radiographs the dental hygienist use slow speed film because this strategy will also reduce scattered radiation. (case F)

NEITHER the statement NOR the reason is correct (case F)

On a radiograph, the inverted Y structure represents the junction between which two areas?

Nasal fossa and maxillary sinus

Nasal pits are formed from the

Nasal placodes

Which condition is most likely to be related to the following symptoms: fetid-odor, punched out papillae, necrotic sloughing tissue, bleeding gums, pain, fever, loss of attachment.

Necrotic ulcerative periodontitis

Which antihypertensive agent is associated with gingival enlargement?

Nifedipine

All of the following drugs use the kidneys as a major pathway of elimination and must be used with caution for this patient EXCEPT one. Which one is the EXCEPTION? (Case C)

Nitrous oxide-oxygen (case C)

Standard radiographic film size for intraoral dental radiography is

No 3 size film - 27x54 mm

The gap between the adjacent Schwann cells is called the

Node of Ranvier

A Mission of Mercy (M.O.M.) event was planned for an urban city in a southern state of the United States. This 2-day event was designed to meet the critical needs of underserved people of all ages by providing free dental care to as many adults and children as time, volunteers, and supplies would allow. Local dentists, dental hygienists, dental assistants, dental students, dental hygiene students, and dental assisting students volunteered for the event. Local politicians and two state representatives were also in attendance. Before receiving dental treatment, informed consent was required of all patients. A 10-question patient survey was distributed, and volunteers reviewed each question with patients while the patients were completing the questionnaire. Confidential questions included information on age, gender, education, family income, address, and past dental experiences. At the end of each patient's treatment and before each volunteer completed his or her shift, the patient was asked to complete an evaluation survey. There was an 88% response rate, and 79% answered that their experience was positive.What type of data is information such as gender and address?

Nominal

At a subsequent recare visit Stage III gingivitis is noted in maxillary posterior right and left molar areas. What radiographic evidence would confirm this finding? (case I)

Normal bone pattern (case I)

A university dental hygiene school received a state-funded grant to improve the oral health status of senior citizens in a rural area of the United States. The funding was for $50,000 over a 1-year period and covered expenses for salaries, equipment, supplies, and education. It was determined that high rates of root caries, root sensitivity, and gum disease were present in this population. The dental hygiene school decided to address these needs by implementing an oral health program in its clinic and to determine the effectiveness of two different types of desensitizing agents on treating root sensitivity.The hypothesis for this study states, "There is no statistically significant difference between desensitizing agent A and desensitizing agent B in the treatment of root sensitivity." What type of hypothesis does this exemplify?

Null

The area-specific advanced periodontal instruments that feature a working end shaped like a tiny circular disc and designed for easy adaptation to furcations and grooves are the

O'Hehir debridement curets

A state-hired full-time public health dentist and dental hygienist are responsible for collecting data on the dental plaque rates of elementary school-aged children. Correlational studies are planned to determine strength of data between genders and among geographical locations of the elementary schools. It was decided to screen children in grades K, 2, 4, and 6 within 25 schools throughout the state. The 25 schools were chosen on the basis of geographic locations and number of students enrolled. All students within each of the chosen grades would be part of the study. Three other public health dental hygienists were hired part time to help in the data collection process. When data collection was completed, analyzed, and interpreted, the public health dentist and the dental hygienist would decide on an oral health program to best meet the needs of the elementary school-aged children of the state.The MOST appropriate index to use to measure the plaque rate of these children is

OHI-S.

Each of the following radiographic exposures provide images of larger areas of the head or skull and require extraoral film placement EXCEPT one. Which one is the EXCEPTION?

Occlusal

Which of the following is NOT a contradiction for application of pit-and-fissure sealants? (case G)

Occlusal contour of tooth (case G)

As the dental hygienist replaces the dental sealant on tooth #14, the sealant dislodges from the tooth when the patient is rinsing. Which of the following factors is MOST likely the cause of the failure of sealant placement? (case G)

Operator error (case G)

Each of the following is contained within cellular cytoplasm EXCEPT one. Which one is the EXCEPTION?

Organelles

Which of the following terms BEST describes the radiopaque lesion in the left maxillary sinus? (case D)

Osteoma (case D)

Vitamin D deficiency in adults can cause which condition?

Osteomalacia (softening of bones) In children, it is called rickets.

Which of the following is the BEST term for the sign of the disease or disorder on tooth #14? (case D)

Overeruption (case D)

The American Dental Hygienists' Association (ADHA) and the Canadian Dental Hygienists' Association (CDHA) have established codes of ethics. These codes encourage all of the following EXCEPT one. Which one is the EXCEPTION?

Overseeing the responsibilities of the dentist

A state-hired full-time public health dentist and dental hygienist are responsible for collecting data on the dental plaque rates of elementary school-aged children. Correlational studies are planned to determine strength of data between genders and among geographical locations of the elementary schools. It was decided to screen children in grades K, 2, 4, and 6 within 25 schools throughout the state. The 25 schools were chosen on the basis of geographic locations and number of students enrolled. All students within each of the chosen grades would be part of the study. Three other public health dental hygienists were hired part time to help in the data collection process. When data collection was completed, analyzed, and interpreted, the public health dentist and the dental hygienist would decide on an oral health program to best meet the needs of the elementary school-aged children of the state.Which of the following P levels associated with a test of significance would BEST indicate that the researcher's results were MOST likely caused by an independent variable rather than by chance occurrence?

P ≥ 0.001

Which bacteria is MOST closely related to chronic periodontitis?

P. gingivalis

Diabetes mellitus is a disease of which of the following organs? (case J)

Pancreas (case J)

A healthy 16-year-old patient has not been to the dentist in two years and has described some throbbing pain in the back of the jaw on both sides. Which type of radiographic images would BEST assess the patient's chief complaint?

Panoramic radiograph

Angular cheilosis is an oral manifestation of a deficiency in each of the following nutrients EXCEPT one. Which one is the EXCEPTION?

Panothenic acid

A Mission of Mercy (M.O.M.) event was planned for an urban city in a southern state of the United States. This 2-day event was designed to meet the critical needs of underserved people of all ages by providing free dental care to as many adults and children as time, volunteers, and supplies would allow. Local dentists, dental hygienists, dental assistants, dental students, dental hygiene students, and dental assisting students volunteered for the event. Local politicians and two state representatives were also in attendance. Before receiving dental treatment, informed consent was required of all patients. A 10-question patient survey was distributed, and volunteers reviewed each question with patients while the patients were completing the questionnaire. Confidential questions included information on age, gender, education, family income, address, and past dental experiences. At the end of each patient's treatment and before each volunteer completed his or her shift, the patient was asked to complete an evaluation survey. There was an 88% response rate, and 79% answered that their experience was positive.All of the following are reasons for the success of the questionnaire data collection process EXCEPT one. Which one is the EXCEPTION?

Participants were given a deadline to complete the questionnaire.

What description is related to the patient who had a spinal injury at C2 and C3 level?

Patient has limited head and neck movement and patient may depend on a ventilator for respiration

All of the following can decrease the effectiveness of a topical anesthetic agent EXCEPT one. Which one is the EXCEPTION?

Patient nervousness

Each of the following positioning errors during panoramic radiography will produce a Frankfort plane error EXCEPT one. Which one is the EXCEPTION? (case D)

Patient's head is not centered (case D)

The full mouth series of radiographs indicate that this patient has experienced all of the following EXCEPT one. Which one is the EXCEPTION? (case F)

Periapical pathology (case F)

A teenage patient comes to the dental office and complains about the pain that radiates through her jaw and neck. Upon examination, the clinician sees inflammation surrounding the crown of a partially erupted third molar. This is most likely to be

Pericoronitis

The periodontal surgery that may be used as a method of surgical curettage is the

Periodontal flap surgery

Sealant material is classified by many parameters including filler content. Filler content classification for pit and fissure sealants includes each of the following types EXCEPT one. Which one is the EXCEPTION

Photo-polymerizing

Emergency care of a patient experiencing an epileptic seizure includes all of the following EXCEPT one. Which one is the EXCEPTION?

Placing a bite-block in the mouth to avoid fracturing of teeth

Each of the following are steps indicated when a curet tip breaks during periodontal debridement EXCEPT one. Which one is this EXCEPTION?

Placing the patient in a supine position

A university dental hygiene school received a state-funded grant to improve the oral health status of senior citizens in a rural area of the United States. The funding was for $50,000 over a 1-year period and covered expenses for salaries, equipment, supplies, and education. It was determined that high rates of root caries, root sensitivity, and gum disease were present in this population. The dental hygiene school decided to address these needs by implementing an oral health program in its clinic and to determine the effectiveness of two different types of desensitizing agents on treating root sensitivity.During which of the following public health process stages is the final development of program goals and objectives accomplished?

Planning

Which indices can help the patient understand the specific areas she needs to focus on with her care at home?

Plaque control record

Aspirin has all of the following pharmacologic effects EXCEPT one. Which one is the EXCEPTION?

Platelet cohesion

All of the following are direct risk factors for this patient's high blood pressure EXCEPT for one. Which one is the EXCEPTION? (case H)

Poor oral self-care (case H)

Which of the following statements is NOT true for a patient experiencing respiratory difficulties? (case J)

Positive pressure oxygen is preferred over rescue breathing (case J)

Which one of the following components is added to dentifrices for its anti-sensitivity quality?

Potassium nitrate or Strontium chloride

This patient admits that she rarely flosses under her posterior bridge because she finds it frustrating and difficult. Which of the following is the best alternative to flossing under her bridge? (case A)

Power-driven irrigating device with an antimicrobial agent (Case A)

The father of this patient asks what the dental office personnel do to ensure that his son does not contract a disease from other patients. Which of the following explains the concept of "standard precautions"? (case G)

Precautions are taken that assume all blood and body fluids are infectious (case G)

A university dental hygiene school received a state-funded grant to improve the oral health status of senior citizens in a rural area of the United States. The funding was for $50,000 over a 1-year period and covered expenses for salaries, equipment, supplies, and education. It was determined that high rates of root caries, root sensitivity, and gum disease were present in this population. The dental hygiene school decided to address these needs by implementing an oral health program in its clinic and to determine the effectiveness of two different types of desensitizing agents on treating root sensitivity.The local dental hygienists' association is planning to provide educational services for the senior citizens about the prevention and treatment of root caries. Which of the following presentation strategies would MOST effectively foster retention of the information?

Presentation of PowerPoint slides

Which teeth are MOST affected when a child has ECC?

Primary maxillary anterior

The x-ray beams that leave the x-ray tube from the tungsten target create

Primary radiation

Which form of the x-ray beam is MOST detrimental to the patient and the operator?

Primary radiation

The amide type of local anesthetic agent includes all of the following EXCEPT one. Which one is the EXCEPTION? (case H)

Procaine (case H)

Which of the following correctly describe conditions associated with cardiac arrhythmias and dysrhythmias? (case I)

Produces aberrant electrical depolarization (case I)

A health care providers' code of ethical conduct that focuses on patient needs, encourages informed consent, and provides for quality care is a referred to as the

Professional code of ethics.

During football season, this patient would benefit from an athletic mouthguard (mouth protector) for all the following reasons EXCEPT one. Which one is the EXCEPTION? (case B)

Promoting mouth protectors to the other players on the team (case B)

When examining this patient's teeth, the dental hygienist notices that the dental sealant on tooth #14 is partially missing. Which is the best course of treatment? (case G)

Re-etching tooth surface and reapplying the dental sealant (case G)

A patient's blood pressure is 184/110 mmHg. You let the patient rest for 5 minutes, and then recheck the blood pressure. If it is still 184/110 mmHg, what should you do?

Refer to a doctor

Each of the following is a reason that periodontal flap procedures are commonly used in surgical procedures EXCEPT one. Which one is the EXCEPTION?

Repositions the soft tissue in a coronal direction

Absolute contraindication for conscious sedation with nitrous oxide - oxygen include all of the following EXCEPT one. Which one is the EXCEPTION?Which one of the following is MOST characteristic of nitrous oxide abuse?

Respiratory infection such as with sinus or tonsils; cold; active allergies; active tuberculosis (TB)

What is the purpose of the lead collimator on radiology equipment? (case J)

Restricts the size and shape of the x-ray beam (case,J)

The impression material agar-agar is a

Reversible hydrocolloid

Which vitamin is the MOST closely associated with angular cheilitis?

Riboflavin

Removal of supragingival or subgingival calculus without intentional removal of tooth surface is referred to as

Scaling

Which of the following treatment considerations is acceptable for patients with myasthenia gravis?

Scheduling short morning appointments

Needle-stick injury is a major cause of disease transmission to health care personnel. Which of the following techniques is BEST to prevent such injury?

Scoop recapping technique

Which dental biomaterials is used to close the pits and fissures on tooth surfaces?

Sealant

If a patient who has suffered congestive hear failure presents with breathing difficulty, what modifications should be made with regard to patient positioning?

Seat patient upright at a 45-degree angle

When a patient is experiencing an asthmatic attack, which position can BEST help with breathing?

Seated position

Which one of the following drugs is NOT a selective serotonin reuptake inhibitor (SSRI) prescribed to reduce anxiety?

Secobarbital (Seconal)

Comparing permanent maxillary first and permanent maxillary second molars, each of the following statements is accurate EXCEPT one. Which one is the EXCEPTION?

Second molars have a minor cusp on the mesiolingual cusp, (cusp of Carabelli)

A state-hired full-time public health dentist and dental hygienist are responsible for collecting data on the dental plaque rates of elementary school-aged children. Correlational studies are planned to determine strength of data between genders and among geographical locations of the elementary schools. It was decided to screen children in grades K, 2, 4, and 6 within 25 schools throughout the state. The 25 schools were chosen on the basis of geographic locations and number of students enrolled. All students within each of the chosen grades would be part of the study. Three other public health dental hygienists were hired part time to help in the data collection process. When data collection was completed, analyzed, and interpreted, the public health dentist and the dental hygienist would decide on an oral health program to best meet the needs of the elementary school-aged children of the state.

Secondary

Caries that appear underneath a restoration are called

Secondary caries

When one drug reduces the absorption of another drug, what is a common strategy to minimize the interaction?

Separating administration of doses by two hours

Pain associated with dentinal hypersensitivity can be described by each of the following EXCEPT one. Which one is the EXCEPTION?

Short duration

Rem represents the measurement for dose equivalent in the Standard system. Rem is equivalent to which measurement in the SI system?

Sievert

Which one of the following patient factors is an indication for a dental implant?

Single missing tooth

Which structure protects the bacterial community and helps biofilm to grow?

Slime layer

In addition to the chipped tooth, which of the following adverse outcomes secondary to tongue piercing is present in this patient? (case F)

Soft tissue injury (case F)

(Case I Images) The gingival tissue around the facial of teeth #23 through #25 is erythematous and bleeds easily on probing. The patient reports that the area has been very tender for the past several months. Which of the following BEST describes what is happening in this area?

Stage III gingivitis (case I)

Which stage of gingivitis marks the transition to periodontitis?

Stage IV

According to the framework established by the 2018 American Association of Periodontology (AAP) guidelines for staging and grading periodontitis, this patient's condition places him in which stage of periodontitis? [Please note: the AAP World Workshop occurred in 2017; guidelines were released in 2018] (case H)

Stage IV (case H)

A state-hired full-time public health dentist and dental hygienist are responsible for collecting data on the dental plaque rates of elementary school-aged children. Correlational studies are planned to determine strength of data between genders and among geographical locations of the elementary schools. It was decided to screen children in grades K, 2, 4, and 6 within 25 schools throughout the state. The 25 schools were chosen on the basis of geographic locations and number of students enrolled. All students within each of the chosen grades would be part of the study. Three other public health dental hygienists were hired part time to help in the data collection process. When data collection was completed, analyzed, and interpreted, the public health dentist and the dental hygienist would decide on an oral health program to best meet the needs of the elementary school-aged children of the state.Which of the following is characteristic of the type of sampling used to collect these data?

Stratified

Which bacteria are mainly involved in caries formation?

Streptococcus mutans

The dentist with whom you work suddenly seems confused and cannot speak. You also notice that her handwriting has changed and she has difficulty using her dominant hand. What medical emergency situation could this be related to?

Stroke

During treatment, the dental hygienist noticed excessive salivation in the patient. Which of the following salivary glands produce the largest amount of saliva in the mouth? (case B)

Submandibular (Case B)

Helix refers to the

Superior margin of the ear

All of the following are prerequisites for caries development EXCEPT one. Which one is the EXCEPTION?

Susceptible host

During which stage of syncope does the patient exhibit flaccid muscles, impaired consciousness, pallor, shallow breathing and a weak/slow pulse? (case I)

Syncope stage (case I)

What type of lymphocyte is thymus-derived and participates in a variety of cell-mediated immune reactions?

T lymphocyte

Which one of the following point of spinal cord injury will allow the patient to use all upper extremities?

T-10

Which of the following vertebrae is affected by the spinal cord injury of this patient with paraplegia? (case F)

T-thoracic (case F)

The term succedaneous applies to which of the following? (case G)

Teeth that replace primary teeth (case G)

All of the following are contraindications for dental sealant application EXCEPT one. Which one is the EXCEPTION?

Teeth with noncavitated lesions

Dentin that is formed in response to injury is referred to as

Tertiary dentin

When taken with antacids, which one of the following can result in a decrease in intestinal absorption of the medication?

Tetracycline

A face shield does not replace the need for other forms of protective eyewear.Face shields do not provide the same respiratory protection as provided by facemasks.

The first statement is FALSE, the second is TRUE

The attraction of new bacteria to previously attached bacteria is referred to as colonization. Extracellular polysaccharides provide structural and energy sources for plaque biofilms.

The first statement is FALSE, the second is TRUE

Tobacco use is not considered a strong environmental risk factor for chronic periodontitis.It has been found that smoking impairs revascularization during healing. (case H)

The first statement is FALSE, the second is TRUE (case H)

Law is divided into three categories — labor, civil and criminal.Criminal law is law established for preventing harm against society and describes a criminal act and its appropriate punishment.

The first statement is FALSE, the second is TRUE.

A Mission of Mercy (M.O.M.) event was planned for an urban city in a southern state of the United States. This 2-day event was designed to meet the critical needs of underserved people of all ages by providing free dental care to as many adults and children as time, volunteers, and supplies would allow. Local dentists, dental hygienists, dental assistants, dental students, dental hygiene students, and dental assisting students volunteered for the event. Local politicians and two state representatives were also in attendance. Before receiving dental treatment, informed consent was required of all patients. A 10-question patient survey was distributed, and volunteers reviewed each question with patients while the patients were completing the questionnaire. Confidential questions included information on age, gender, education, family income, address, and past dental experiences. At the end of each patient's treatment and before each volunteer completed his or her shift, the patient was asked to complete an evaluation survey. There was an 88% response rate, and 79% answered that their experience was positive.Two types of program evaluation are formative and performance.Formative, or ongoing, evaluation is a continual process throughout the program that can lead to revisions that improve program success.

The first statement is TRUE, the second is FALSE

The pathological lesion evident between the roots of tooth #8 and tooth #9 is referred to as a nasopalatine duct cyst.A globulomaxillary cyst is located between the maxillary central incisors. (case I)

The first statement is TRUE, the second is FALSE (case I)

Minimizing the recurrence and progression of periodontal disease is an objective of periodontal maintenance. The recommended frequency of periodontal maintenance care visits decrease when the self-care practices of patients are less than optimal.

The first statement is TRUE, the second is FALSE.

Tachycardia is characterized by an increase in pulse rate.Bradycardia is characterized by premature impulses resulting in premature atrial beats

The first statement is TRUE, the second is FALSE.

A total cholesterol level of less than 200 mg/dL is a desirable level that places this patient at lower risk for coronary artery disease. Low-density lipoprotein (LDL) levels decrease the risk of heart disease, and high-density lipoprotein (HDL) levels increase the risk. (case D)

The first statement is TRUE, the second is FALSE. (case D)

Horizontal bone loss is evident between teeth #20 and #21.The type of pocket formed between teeth #20 and #21 is referred to as an infrabony pocket. (case E)

The first statement is TRUE, the second is FALSE. (case E)

Condyloma acuminatum is caused by the herpes virus. Lesions of condyloma acuminatum appear pink and papillary.

The first statement is false; the second is true

Amperage is the force that moves the electrons in a electrical circuit. Higher amperage indicates that more electrons are present.

The first statement is false; the second is true. Voltage is the force that moves the electrons.

Overconsumption of most nutrients can be detrimental to the body. However, overconsumption of vitamins is always beneficial.

The first statement is true; the second is false

The cutting edges of scaling instruments dull with repeated sterilization. Sharpening stones cannot be sterilized.

The first statement is true; the second is false

The Naber's probe is designed specifically for the detection of furcations. The Naber's probe is single-ended.

The first statement is true; the second is false.

When planning an educational session for this patient, which of the following concepts should be included? (case J)

The presence of infection may make diabetes more difficult to regulate (case J)

A dental hygienist recognizes all relationships require considering the values and perspectives of others before making decisions or taking actions affecting them. Which fundamental principles of the Code of Ethics for Dental Hygienists is this hygienist MOST embracing?

The principle of complementarity recognizes that in all relationships, it requires considering the values and perspectives of others before making decisions or taking actions affecting them.

Which component of the radiographic developer solutions reduces the exposed silver halide crystals to black metallic silver?

The reducing agent (hydroquinone and elon)

Sustained-release, local drug delivery systems used in conjunction with periodontal debridement for pocket reduction is preferred over systemic delivery because local delivery occurs immediately and does not require prolonged time periods.

The statement is correct, but the reason is NOT

Dysphagia is often associated with chronic alcoholism because xerostomia causes difficulty breathing.

The statement is correct, but the reason is NOT.

Older adults are at an increased risk for root surface caries (RSC) because this demographic group typically consumes a high protein diet.

The statement is correct, but the reason is NOT.

Which bones form the lateral walls of the skull?

The temporal bones

The cheekbone is composed of the

The temporal process of the zygomatic bone and the zygomatic process of the temporal bone

Which of the following does NOT describe the process of energy storage and transfer, as it occurs during microbial metabolism and cell regulation?

The term catabolism describes biosynthetic reactions that use energy.

What describes proper positioning of the tongue during a panoramic radiograph?

The tongue should be touching the roof of the palate

The patient returns 4 weeks after nonsurgical periodontal therapy. Which of the following is the single BEST criterion for evaluating the success of this treatment?

Tissue health

What is the purpose of beta blockers?

To decrease nerve impulses to the heart and blood vessels These medications end with the suffix -olol, as in atenolol (Tenormin).

To provide the most optimal treatment to this patient, it is the responsibility of the dental hygienist to make all of the following accommodations EXCEPT one. Which one is the EXCEPTION? (case F)

To take the responsibility of transferring the patient from the wheelchair to the dental chair (case F)

A patient who decides to sue a dentist for improper placement of an amalgam restoration is engaging in which legal area?

Tort law

Which microorganism causes syphilis?

Treponema pallidum

The dental hygienist would like to recommend a self-care product that specifically combats gingivitis through antimicrobial action. Which active ingredient in the product would BEST provide this benefit? (case D)

Triclosan (case D)

What observation type uses a mouth mirror, an explorer, and a light?

Type III

Which type of hypersensitivity has the MOST delayed response?

Type IV

Which of the following is the BEST choice to remove subgingival calculus for this patient? (case C)

Ultrasonic scaling instrument (case C)

The muscles of facial expression and taste within the tongue are controlled by cranial nerve

VII (facial nerve)

Which vitamin is important for night vision?

Vitamin A

Which vitamin deficiency during pregnancy can cause spina bifida?

Vitamin B9 (folate)

Which bone forms the posterior part of the nasal septum?

Vomer

When a patient is affected by cyclic neutropenia, when should the clinician treat the patient?

When the neutrophil count is normal

The ideal probe for detecting and assessing bone loss in a furcation area has all of the following characteristics EXCEPT one. Which one is the EXCEPTION? (case H)

Wide toe (case H)

Injury below the T (thoracic) level

Will allow the normal motor function of the upper body.

Injury at the C5 level

Will allow the patient to have shoulder control and perform daily tasks of eating.

Injury at the C7 level

Will allow the patient to have wrist and elbow movements as well.

The time elapsed from the point of mixing a material until its initial setting is known as?

Working time

Which of the following forms of electromagnetic radiation has the shortest wavelength?

X-rays and gamma rays

Which one of the alternative sweeteners is a nutritive sweetener?

Xylitol

In the fertilization stage of the preimplantation period, the sperm and ovum join to form,

Zygote

Hemorrhagic stroke

a blood vessel ruptures in the brain.

How frequently can nitroglycerin be administered for the treatment of angina? a. 1 tablet every 5 minutes, not to exceed 3 tablets in 15 minutes b. 1 tablet every 10 minutes, not to exceed 3 tablets in 30 minutes c. 2 or 3 tablets every 5 minutes d. 2 or 3 tablets every 10 minutes

a. 1 tablet every 5 minutes, not to exceed 3 tablets in 15 minutes One tablet every 5 minutes should be given to your client if the angina episode doesn't subside; do not exceed more than three tablets in 15 minutes.

Which ASA level involves a healthy, non-smoking individual with little to no alcohol use?

a. ASA I

Which of the following characterizes a complete protein? a. Adequate in all the essential amino acids b. Synthesized by the body c. Provided through plant foods d. A dispensable amino acid

a. Adequate in all the essential amino acids Animal proteins are considered complete (high quality) proteins. Composition of human tissue more closely resembles that of animal tissue than that of plant tissue. Tissue similarities enable the human body to use animal proteins MORE effectively for its main functions: body maintenance, repair, growth. Animal proteins contain all the essential amino acids in amounts that are sufficient for the body.

What is the best recommendation that the dental hygienist can give to a client to reduce her risk for developing recurrent oral fungal infections? a. Advise the client to rinse twice daily with an antiseptic mouth rinse. b. Instruct the client to change toothbrushes frequently. c. Teach the client not to share toothpaste or cosmetics. d. Encourage the client to chew xylitol gum preventively.

a. Advise the client to rinse twice daily with an antiseptic mouth rinse. Fungal infections must be treated with the appropriate prescription antifungal medication. To prevent recurrent fungal infections, the client should be instructed to dispose of any contaminated items, such as toothbrushes and cosmetics, and to rinse twice daily with a broad-spectrum antimicrobial mouth rinse. Both 0.12% chlorhexidine gluconate and essential oil mouth rinses kill seven species of Candida, the most frequent organism associated with intraoral fungal infections, without promoting resistance among the resident oral bacteria or the development of opportunistic infections. Xylitol has no effect on fungal organisms.

Which of the following provides the most conclusive diagnostic evidence in distinguishing pemphigus from pemphigoid? a. Biopsy and histology report b. Clinical picture c. Nikolsky's sign d. History of the disease

a. Biopsy and histology report The histology report is most significant because in pemphigus, the lesion is intraepithelial, but in pemphigoid, the lesion is subepithelial; Clinically, the lesions may be very similar; History of the disease is noncontributory because both are autoimmune conditions; Nikolsky's sign can be present in both.

What is the most common site for cancer in women? a. Breast b. Uterus c. Colon d. Lungs

a. Breast The most common site for malignancy (cancer) in women is the breast. In men, the top three - lung, prostate, and colorectal cancers - contributed 44.4% of all cancers (excluding non-melanoma skin cancer).

Case A Isabelle Sanchez, a registered dental hygienist (RDH), has been working in the same practice, substituting for a hygienist who is on maternity leave. Today, when she reports to work, Isabelle is told by the dental assistant that the elderly man whom Isabelle treated last Monday is in the hospital and has been diagnosed with Legionnaires' disease. During the last week, he had only left his house to go to his dental visit, and the water tested in his house was negative for Legionella. In hindsight, Isabelle realizes that in her treatment room, she did not see a water bottle attached to the master control unit that would have indicated the use of an independent water system. She just assumed that the dental unit water lines were being treated with another method. What was the portal of entry of Legionella into the elderly patient's body? a. Breathing in contaminated water vapor b. Swallowing contaminated water c. Splashing of droplets on mucous membranes d. Infiltrating microbes into nonintact mucosa

a. Breathing in contaminated water vapor

Increased blood flow and the release of inflammatory mediators cause which cardinal sign of inflammation? a. Calor b. Dolor c. Rubor d. Functio laesa

a. Calor Increased blood flow and the release of inflammatory mediators cause calor; Rubor is caused by increased vascularity; Dolor is caused by the stretching of pain receptors and nerves by inflammatory exudates and the release of chemical mediators; Function laesa is caused by a combination of physiologic effects of inflammation.

Which of the following produce(s) no radiographic image? a. Fibroma b. Supernumerary teeth c. Dental caries d. Odontoma

a. Fibroma A fibroma is composed of soft tissue and cannot be seen radiographically; Dental caries appear radiolucent; Supernumerary teeth appear radiopaque; An odontoma appears radiopaque with a radiolucent halo; the two types of odontoma are compound and complex. Each one has a unique radiographic appearance.

Which of the following is correct concerning the condition hypokalemia? a. Lack of potassium b. Restriction of the consumption of aspartame c. Ineffective metabolism of phenylalanine to threonine d. Lack of calcium

a. Lack of potassium Lack of potassium causes hypokalemia. Lack of calcium would cause osteopenia and osteoporosis. The liver cannot metabolize essential amino acid phenylalanine into nonessential amino acid tyrosine with phenylketonuria (PKU).

Which of the following allows lipids to be transported in the blood? a. Lipoproteins b. Bile c. Phospholipids d. Water

a. Lipoproteins Lipoproteins are a means of transportation through the bloodstream for cholesterol, triglycerides, and phospholipids. Lipoproteins include chylomicrons, very low-density lipoproteins, low-density lipoproteins, and high-density lipoproteins.

Where does the majority of drug metabolism occur in the human body? a. Liver b. Intestines c. Kidney d. Blood

a. Liver The vast majority of drugs are metabolized by the liver via the hepatic microsomal cytochrome P450 enzyme system. Some metabolism may also occur in the kidneys, intestinal tract, and plasma. Most drugs in dentistry are metabolized by the liver.

Which of the following can be a characteristic of pemphigus vulgaris? a. Nikolsky sign b. Is caused by a drug reaction c. Occurs predominately in African American females d. Occurs predominately in white males

a. Nikolsky sign Nikolsky sign is a condition in which an air syringe or tongue blade can be used to manipulate the epithelium, creating a blister-like lesion; No gender or racial predilection exists in pemphigus vulgaris; Pemphigus vulgaris is a severe progressive autoimmune disease; No gender or racial predilection exists in pemphigus vulgaris.

Where should you place your hands when compressing an infant's chest during CPR? a. One hand on the infant's forehead and two or three fingers on the center of the chest b. One hand on the forehead and one hand on the chest c. One hand on the chin and two or three fingers on the center of the chest d. One hand on the chin and one hand on the chest

a. One hand on the infant's forehead and two or three fingers on the center of the chest Hand positions when compressing an infant's chest during CPR are one hand on the infant's forehead and two or three fingers of the other hand on the lower half of the chest (one finger width below nipple line).

The pleomorphic adenoma is most often found in the: a. Palate b. Lymph nodes c. Buccal mucosa d. Mandible

a. Palate The palate is the most common location for pleomorphic adenoma; The mandible is not an appropriate location for pleomorphic adenoma; The buccal mucosa is not the usual location for pleomorphic adenoma; Lymph nodes are not common locations for pleomorphic adenoma.

What field studies the absorption, distribution, metabolism, and excretion of drugs? a. Pharmacokinetics b. Pharmacodynamics c. Pharmacotherapeutics d. Pharmacogenomics

a. Pharmacokinetics Pharmacokinetics describes how a drug is absorbed, distributed, metabolized, and eventually excreted from the body. Pharmacodynamics is the study of drug actions in the body, meaning how drugs interact with receptors and produce their therapeutic effects. Pharmacogenomics is the study of how genetic variations can alter pharmacokinetics and pharmacodynamics. Pharmacotherapeutics is the study of using drugs to produce effects in the body.

Which of the following phases of a seizure poses the greatest risk for a client? a. Postictal b. Ictal c. Prodromal d. All phases pose equal risk.

a. Postictal The most dangerous phase is the postictal phase.

All of the following conditions are examples of negative nitrogen balance EXCEPT one. Which one is the exception? a. Pregnancy b. Blood loss c. Infection d. Anorexia nervosa

a. Pregnancy Nitrogen balance refers to balance between input and output of nitrogen. If the body is NOT growing or in need of extra protein to recover from illness or infection, only enough protein is needed to equal input to output. Negative protein balance exists when output is greater than input (as occurs with semistarvation, infection, fever, blood loss). However, if the body is growing, as during childhood, pregnancy, or recovery from illness, MORE protein will be necessary to supply the body's needs for tissue repair. Increased need for protein intake is defined as positive protein balance.

Endogenous chemoattractants involved in chemotaxis include all of the following EXCEPT: a. SIRS b. LTB4 c. C5a d. IL-8

a. SIRS SIRS refers to the systemic inflammatory response syndrome; it is not an endogenous chemoattractant involved in chemotaxis; C5a is an endogenous chemoattractants involved in chemotaxis; LTB4 is an endogenous chemoattractants involved in chemotaxis; IL-8 is an endogenous chemoattractants involved in chemotaxis.

All of the following organs are responsible for drug excretion except for one. What is the EXCEPTION? a. Spleen b. Saliva c. Kidney d. Lungs

a. Spleen

In which stage of gingivitis do PMNs attach to the vessel walls and begin to migrate into the surrounding connective tissues? a. Stage I Gingivitis (Initial/Subclinical Stage) b. Stage II Gingivitis (Early Stage) c. Stage III Gingivitis (Established Stage) d. Stage IV Gingivitis (Advanced Stage)

a. Stage I Gingivitis (Initial/Subclinical Stage)

Which term is used for the practice of treating a patient's blood, body fluids, nonintact skin, and mucous membranes as potentially infectious? a. Standard precautions b. Pervasive precautions c. Universal precautions d. Protective precautions

a. Standard precautions

Nitrous oxide-oxygen analgesia is associated with which of the following adverse drug effects? a. Teratogenicity b. Seizures c. Nephrotoxicity d. Liver failure

a. Teratogenicity Exposure to nitrous oxide gas was linked to spontaneous abortion in pregnant dental office workers during the 1970s before the use of scavenging systems. Use of a scavenger device significantly decreases the amount of gas that escapes into the room air, which dramatically reduces this risk. Clients undergoing nitrous oxide sedation should be encouraged to breathe through their noses, not their mouths, to further minimize the amount of gas that is exhaled.

Mineral oils interfere with the absorption of which of the following vitamins? a. Vitamin A b. Vitamin C c. Vitamin B6 d. Vitamin B12

a. Vitamin A Vitamin A is a fat-soluble vitamin, and others are water soluble. Mineral oils interfere with the absorption of fat-soluble vitamins.

Which of the following is the best food source of magnesium? a. Whole grains and nuts b. Dark yellow vegetables c. Citrus fruits d. Dairy products

a. Whole grains and nuts Magnesium facilitates the operation of enzymes and assists with the relaxation of muscles after contraction. Foods rich in magnesium include dark green leafy vegetables, whole-grain breads and cereals, nuts, legumes, and seafood.

Which Food and Drug Administration (FDA) Pregnancy Category represents the highest degree of risk to the fetus? a. X b. C c. B d. A

a. X The FDA Pregnancy Categories reflect the relative risk to the fetus on exposure to a drug. A rating of X represents the highest degree of teratogenic risk to the developing fetus.

Which of the following adverse drug effects is the most common oral complication associated with medication use? a. Xerostomia b. Taste alteration c. Caries d. Periodontal disease

a. Xerostomia Xerostomia is the most common oral manifestation of medication use. It is also the most common oral complaint among elderly clients, who often take multiple medications that cause this side effect. Xerostomia may contribute to oral infections, including caries and progression of periodontal disease. Salivary changes can also contribute to taste alteration, although this adverse reaction is often caused by drug excretion into the saliva.

The cardinal signs of inflammation include all of the following, EXCEPT one. Which one is the EXCEPTION? a. Odor b. Heat c. Redness d. Swelling e. Pain

a. odor Odor is NOT one of the cardinal signs of inflammation, which include heat, redness, pain, swelling, loss of function.

All the following management procedures should be utilized to prevent an angina attack for a client with a history of angina EXCEPT a. use local anesthetic for pain control; maximum 4 carpules of 1:100,000 epinephrine. b. administer low-flow oxygen. c. short appointments scheduled in the late morning or early afternoon. d. do not see a client with unstable angina, and refer to his or her physician.

a. use local anesthetic for pain control; maximum 4 carpules of 1:100,000 epinephrine. A maximum of two carpules of local anesthetic containing 1:100,000 epinephrine can be given for pain control. Short, late morning or early afternoon appointments should be scheduled. You should refer clients with unstable angina to their physician before treating them. Low flow oxygen should be administered at 4 to 6 L/min.

The interdental col area is particularly susceptible to destruction because of

absence of keratinization and interproximal location

The patient reports that results of a preprandial blood glucose test two weeks earlier were 120 mg/dL. Which of the following BEST describes the interpretation of this test? (case J)

acceptable (case J)

A tooth desensitizing agent is applied continually

according to manufacturer's instruction

A state-hired full-time public health dentist and dental hygienist are responsible for collecting data on the dental plaque rates of elementary school-aged children. Correlational studies are planned to determine strength of data between genders and among geographical locations of the elementary schools. It was decided to screen children in grades K, 2, 4, and 6 within 25 schools throughout the state. The 25 schools were chosen on the basis of geographic locations and number of students enrolled. All students within each of the chosen grades would be part of the study. Three other public health dental hygienists were hired part time to help in the data collection process. When data collection was completed, analyzed, and interpreted, the public health dentist and the dental hygienist would decide on an oral health program to best meet the needs of the elementary school-aged children of the state.The evaluation of a dental health program implemented in these elementary schools should be PRIMARILY concerned with

accuracy of collected data, its analysis, and its interpretation.

Potassium sulfate

acts as an accelerator for the gypsum material.

For a topical antimicrobial mouthrinse to claim that it has "substantivity" the product must be able to

adhere to soft and hard tissue for a long duration while releasing active ingredient(s).

Water sticking to other substances demonstrates the principle of

adhesion

Agents such as sodium bicarbonate and chlorine dioxide are added to oral self-care products to

alleviate oral malodor.

Gas exchange during respiration takes place in the (case J)

alveoli (case J)

Nitroglycerin is used to treat patients experiencing

angina pectoris.

The surface of the condyloid process of the mandible articulates with the

articular disk of the TMJ

The dark lines evident on the left premolar bitewing and left mandibular molar periapical exposures are MOST likely caused by (case H)

artifacts (case H)

Torus palatinus, an inherited, autosomal dominant condition appears

as a variety of shapes - nodular, lobulated, smooth (flat), spindle.

Drug interactions can occur between methotrexate (MTX) and (case E)

aspirin (case E)

The incisal edges of the anterior mandibular teeth are characteristic of (case A)

attrition (case A)

Periodontitis can be categorized by the number of sites affected. Which of these percentages indicates the cut-off above which periodontitis can be categorized as generalized periodontitis? a. 20% b. 30% c. 40% d. 50% e. 60%

b. 30% Generalized periodontitis is defined as periodontitis involving >30% of sites.

Your patient is a 5'9" male weighing 220 lbs with a BMI of 32.5. He does not smoke and is in fairly good health besides having seasonal allergies. The patient does not smoke but occasionally drinks in social situations. Which ASA level does the patient fall into? a. ASA I b. ASA II c. ASA III d. ASA IV e. ASA V f. ASA VI

b. ASA II The classification of ASA II is defined by "Mild diseases only without substantive functional limitations. Current smoker, social alcohol drinker, pregnancy, obesity (30<BMI<40), well-controlled DM/HTN, mild lung disease." Since the patient has allergies, a BMI indicating obesity, and consumes alcohol, he fits into this category. ASA I means that a patient is healthy, does not smoke, and consumes little to no alcohol, and ASA III means that a patient has a severe systemic disease, such as morbid obesity (greater than or equal to BMI 40).

One of the following characteristics is NOT true concerning calcium. Which one is the EXCEPTION? a. Vitamin D assists in its absorption b. An intake of 400 mg/day is recommended c. Assists in muscle contraction and relaxation d. Aids in coagulation

b. An intake of 400 mg/day is recommended Although the major function of calcium is to form and maintain bones, it has other important functions. It aids in the contraction of muscles and in the blood coagulation process. It also may reduce the risk of colon cancer. Most of its absorption is in the upper intestine, with the assistance of active vitamin D hormone. The recommended daily allowance of both calcium and phosphorous is 1000 mg/day.

Case A Isabelle Sanchez, a registered dental hygienist (RDH), has been working in the same practice, substituting for a hygienist who is on maternity leave. Today, when she reports to work, Isabelle is told by the dental assistant that the elderly man whom Isabelle treated last Monday is in the hospital and has been diagnosed with Legionnaires' disease. During the last week, he had only left his house to go to his dental visit, and the water tested in his house was negative for Legionella. In hindsight, Isabelle realizes that in her treatment room, she did not see a water bottle attached to the master control unit that would have indicated the use of an independent water system. She just assumed that the dental unit water lines were being treated with another method. Which of the following microorganism types is responsible for Legionnaires' disease? a. Fungus b. Bacterium c. Virus d. Prion

b. Bacterium

What is the most common drug found in over-the-counter (OTC) products applied topically for pain control? a. Lidocaine b. Benzocaine c. Carbocaine d. Tetracaine

b. Benzocaine Although all of the product choices listed in this question are used for pain control in dentistry, benzocaine in a 20% concentration is most often used in OTC topical pain control formulations. Tetracaine is also found in OTC medications, as well as in professionally applied topical anesthetics.

Which of the following is TRUE regarding the cellular response to injury? a. Response to injury is the same in all cell types. b. Cell necrosis will occur if cell membrane integrity is compromised. c. Cell necrosis will occur only if the cellular genetic apparatus is damaged. d. Duration of an injury has little impact on the cellular response to injury.

b. Cell necrosis will occur if cell membrane integrity is compromised. Cellular response to injury depends on multiple factors, including the type, duration, and severity of the injury; cell type; and biochemical considerations such as cell membrane, genetic apparatus, protein integrity.

Clients with an allergy to penicillin are more likely to show a cross-sensitivity reaction to which of the following classes of antibiotics? a. Tetracyclines b. Cephalosporins c. Macrolides d. Fluoroquinolones

b. Cephalosporins Cross-sensitivity reactions are allergic reactions to one or more drugs with similar chemical structures. When a client experiences a hypersensitivity to one drug in a given class, that client will also be allergic to all of the drugs within the same class. Clients may also experience cross-sensitivity reactions to families of drugs with similar structures, such as the penicillins and the cephalosporin antibiotics.

Clients who have demonstrated weight loss may experience which of the following oral complications? a. Difficulty chewing b. Difficulty wearing dentures c. Difficulty speaking d. Difficulty swallowing

b. Difficulty wearing dentures A minor weight loss can cause difficulty with wearing dentures in an elderly client, resulting in movement and rubbing of the denture during function. Difficulty with speaking, chewing, and swallowing is associated with xerostomia, which may also make wearing dentures or appliances uncomfortable.

What is the most common cause of angular cheilitis? a. Localized dermatitis b. Fungal infection c. Allergic reaction d. Vitamin deficiency

b. Fungal infection Although angular cheilitis may be caused by vitamin B and iron deficiencies, bacterial infections, and the chronic habit of licking the corners of the mouth, the majority of cases are fungal in nature. The organism Candida albicans is most associated with oral fungal infections. Angular cheilitis is treated with a topical antifungal medication combined with a topical steroid in a prescription ointment known as Mycolog II (nystatin with triamcinolone).

Your client has had a cerebrovascular accident (CVA). How long before his next dental hygiene appointment should you have him discontinue his anticoagulant? a. 4 days b. He should continue to take his anticoagulant. c. 2 days d. The morning of the appointment

b. He should continue to take his anticoagulant. The client should continue to take the anticoagulant; discontinuation puts him at greater risk for a CVA.

What is the term used to describe a client who experiences a greater than expected reaction to a drug as compared with a normal reaction? a. Drug idiosyncrasy b. Hypersusceptibility c. Biologic variation d. Tachyphylaxis

b. Hypersusceptibility A client who experiences a desired drug effect at a dose that is lower than the normal dose is said to be hypersusceptible to the drug. This client will require a lower dose of the drug to achieve the desired therapeutic effect and to prevent drug overdose.

Case B After dismissing his last patient of the day, Preston Miller, RDH, returns to his treatment room. He is in a rush because his daughter's softball team has a game tonight and he is the coach. Preston hurriedly closes his instrument cassette, not taking care to arrange the instruments beforehand. In the instrument-processing area, he lifts the cassette off the metal tray to place it into the ultrasonic unit. Unknowingly, an explorer tip is protruding from a hole in the cassette and penetrates his finger through the exam glove. He removes his gloves and sees a small amount of blood forming on his index finger. Additionally, Preston has received the HBV vaccine and has verified immunity to HBV. According to OSHA, which of the following does NOT need to be included in a dental office's record keeping? a. Employee vaccine declination statements b. Log of employee injuries and illnesses c. Records of annual staff trainings on infection control d. Employees' social security numbers

b. Log of employee injuries and illnesses

What is the first white blood cell to emigrate to the site of injury? a. Monocyte b. Neutrophil c. Macrophage d. Eosinophil

b. Neutrophil The neutrophil is the first white blood cell to emigrate to a site of injury; The monocyte is the second white blood cell to emigrate to injured tissue; Eosinophils are seen in immune reactions and parasitic infections; Monocytes become macrophages after emigrating to injured tissue.

Case B After dismissing his last patient of the day, Preston Miller, RDH, returns to his treatment room. He is in a rush because his daughter's softball team has a game tonight and he is the coach. Preston hurriedly closes his instrument cassette, not taking care to arrange the instruments beforehand. In the instrument-processing area, he lifts the cassette off the metal tray to place it into the ultrasonic unit. Unknowingly, an explorer tip is protruding from a hole in the cassette and penetrates his finger through the exam glove. He removes his gloves and sees a small amount of blood forming on his index finger. Additionally, Preston has received the HBV vaccine and has verified immunity to HBV. The dental hygienist elects to be tested for bloodborne pathogens (HCV and HIV), and he is informed of the results by a qualified health care professional. Which of the following actions is he then required to take? a. Document the results in his employee file b. None of the above is required c. Inform the CDC and OSHA d. Inform his employer of the results

b. None of the above is required

Which of the following cysts has the potential for developing into ameloblastoma? a. Residual cyst b. Primordial cyst c. Lateral periodontal cyst d. Stafne bone cyst

b. Primordial cyst

In which stage of gingivitis does BOP usually occur? a. Stage I Gingivitis (Initial/Subclinical Stage) b. Stage II Gingivitis (Early Stage) c. Stage III Gingivitis (Established Stage) d. Stage IV Gingivitis (Advanced Stage)

b. Stage II Gingivitis (Early Stage)

In which stage of gingivitis does the destruction of collagen and fibroblasts begin to occur? a. Stage I Gingivitis (Initial/Subclinical Stage) b. Stage II Gingivitis (Early Stage) c. Stage III Gingivitis (Established Stage) d. Stage IV Gingivitis (Advanced Stage) e. None

b. Stage II Gingivitis (Early Stage)

Which of the following is the major gland that affects the BMR? a. Hypothalamus b. Thyroid c. Sebaceous d. Endocrine

b. Thyroid Major gland that affects the BMR is the thyroid gland. Higher thyroid hormone levels increase the metabolic rate.

What term is used to describe the need for an increasingly larger dose to produce the desired clinical effect? a. Tachyphylaxis b. Tolerance c. Drug idiosyncrasy d. Potency

b. Tolerance The need to take increasingly larger doses of a drug to produce the desired therapeutic effect is termed drug tolerance. Tolerance is a mechanism that often leads to drug dependency and addiction.

Case B After dismissing his last patient of the day, Preston Miller, RDH, returns to his treatment room. He is in a rush because his daughter's softball team has a game tonight and he is the coach. Preston hurriedly closes his instrument cassette, not taking care to arrange the instruments beforehand. In the instrument-processing area, he lifts the cassette off the metal tray to place it into the ultrasonic unit. Unknowingly, an explorer tip is protruding from a hole in the cassette and penetrates his finger through the exam glove. He removes his gloves and sees a small amount of blood forming on his index finger. Additionally, Preston has received the HBV vaccine and has verified immunity to HBV. What should Preston the dental hygienist do next? a. Scrub the finger using a stiff brush b. Wash the finger with antimicrobial soap c. Immerse the finger in bleach for 1 minute d. Squeeze the finger to let out the contaminants

b. Wash the finger with antimicrobial soap

Several nutrients have an effect on the cells of the oral tissues. Which of the following nutrients assists in speeding up wound healing? a. Foliate b. Zinc c. Phosphorus d. Calcium

b. Zinc Adequate zinc intake is essential for the proper development of sexual organs and bone. It also is needed for DNA and protein metabolism and assists with wound healing.

A tumor biopsy is analyzed microscopically. Which of the following pathological findings is most highly associated with a malignant neoplasm? a. Atypia b. Basement membrane invasion c. Increased nuclear/cytoplasmic ratio d. Necrosis e. Pleomorphism

b. basement membrane invasion Basement membrane invasion is a characteristic exclusive to malignant neoplasms. Once a tumor shows invasion beyond the basement membrane or into the vascular system the tumor is automatically considered malignant. Once the tumor metastasizes, the patient's prognosis becomes significantly worse. If the tumor is found to be metastatic to another organ, then the tumor is considered to be a Stage 4 cancer and generally carries a poor prognosis, depending on the cancer.

One of the universal features of a nitrous oxide delivery system is that all of the dials, hoses, and gauges associated with the flow of oxygen are blue. Conversely, all of the equipment used to house the nitrous oxide are green. a. both true b. both false c. first true, second false d. first false, second true

b. both false Even though there are numerous types and manufacturers of nitrous oxide systems, there are some built-in safety similarities. One of the features they all share is the color distinction between the oxygen (green) and nitrous (blue). The reason for this is to eliminate the possibility of mixing up the tanks or hoses. Also, the tubes and fittings are different sizes for the oxygen compared with the nitrous so that the hoses are not interchangeable. Additionally, there is a pin system on the yoke where the gas cylinders are connected to the machine. The pins and holes are spaced differently for oxygen than for nitrous.

Oxygen is used for all the following emergencies EXCEPT a. angina. b. hyperventilation. c. syncope. d. shock.

b. hyperventilation.

ends with olol Ex: Atenolol

beta blocker

The pancreas contains enzymes that digest each of the following fluids EXCEPT one. Which one is the EXCEPTION?

bile

thrombotic stroke (ischemic)

blood clots form in the arteries that supply the brain.

Embolic stroke

blood clots or other debris travel through the blood stream and end up lodging in narrower brain arteries.

The fold of tissue that attaches the lip to the oral mucosa below the mandibular central incisors is the lingual frenum.The incisive papilla is located on the posterior part of the soft palate. (case I)

both statements are false (case I)

Simvastatin, a lipid lowering drug, belongs to the class of pharmaceuticals called statins.The patient should avoid grapefruit when taking simvastatin due to the risk of toxicity (case I)

both statements are true (case I)

During an oral assessment, both the appearance and the ability of Stensen's ducts to function adequately are examined while palpating the

buccal mucosa

How long after a dose of nitroglycerin should angina pain lessen? a. 10 to 15 minutes b. 5 to 8 minutes c. 2 to 4 minutes d. Instantly

c. 2 to 4 minutes Angina pain should decrease within 2 to 4 minutes; there shouldn't be a need for a second or third dose of nitroglycerine. Nitroglycerine does not act instantly. Angina pain should decrease before the need for a second dose of nitroglycerine which is taken 5 minutes after the first dose. Angina pain should decrease long before the third dose of nitroglycerine is needed.

Pituitary hyperfunction is a characteristic of which of the following diseases? a. Cushing's syndrome b. Graves' disease c. Acromegaly d. Diabetes insipidus

c. Acromegaly Gigantism and acromegaly occur when the pituitary gland oversecretes growth hormone. Diabetes insipidus is caused by pituitary hyposecretion, Graves' disease is caused by hypersecretion of the thyroid. Cushing's syndrome is caused by hypersecretion of the adrenal glands.

Which of the following indicators is used to determine sterility? a. Chemical indicator b. External indicator c. Biologic indicator d. Integrated indicator

c. Biologic indicator

Which of the following classes of medications can be used to stimulate salivary flow in patients with functional gland tissue? a. Sympathomimetics b. Anticholinergics c. Cholinergic agonists d. Adrenergic blockers

c. Cholinergic agonists Salivary flow is normally stimulated by neurotransmitters of the autonomic nervous system and is secreted as the first digestive enzyme in the gastrointestinal tract. Drugs that stimulate salivary flow mimic the actions of acetylcholine in the parasympathetic nervous system and are known as cholinergic agonists. An example is the drug pilocarpine (Salagen), which is indicated for the treatment of xerostomia associated with Sjögren's syndrome and head and neck radiation therapy.

Which of the following distinguishes a maxillary first molar from a maxillary second molar? a. fewer mesial marginal ridge tubercles b. less root flare c. Cusp of Carabelli d. the distal marginal ridge is located more cervically than the mesial marginal ridge e. the distobuccal root is narrow so the wider mesiobuccal root is visible from the distal view.

c. Cusp of Carabelli The maxillary first molar is the only molar to have a cusp of Carabelli.

All of the following are normal physiologic changes associated with aging that alter drug activity in the body except which one? a. Decreased plasma proteins b. Decreased liver function c. Decreased body fat d. Decreased kidney function

c. Decreased body fat Liver and kidney functions decline as a natural part of physiologic aging. Other normal physiologic changes associated with aging that alter drug function include increased body fat and decreased lean body muscle, decreased total body water, and increased stomach acidity. With age the body produces fewer plasma proteins, which decreases the availability of binding sites in the circulation, allowing for more free (or active) drug to leave the circulation to bind to target tissues.

All of the following are common pre-existing conditions that may limit the use of certain medications EXCEPT one. Which one is the EXCEPTION? a. Hypertension b. Enlarged prostate c. Duodenal disease d. Thyroid disease

c. Duodenal disease Clients may not be aware of the presence of many common pre-existing conditions before taking over-the-counter or prescription drugs. Furthermore, they may read the labels of these medications and not understand the nature of these conditions. Common pre-existing conditions include thyroid disease, hypertension, and enlarged prostate in men, all of which are often overlooked or underdiagnosed in the general population.

Which of the following lingual papillae has desquamation and is associated with geographic tongue? a. Foliate b. Fungiform c. Filiform d. Circumvallate

c. Filiform Migratory, reversible desquamation of the filiform lingual papillae is noted with geographic tongue (benign migratory glossitis). Name is due to condition resembling a map.

Which of the following best describes manufacturers' recommendations for pediatric drug dosing? a. One-quarter of the adult dose b. Equivalent dosing to the adult dose c. Half of the adult dose d. Twice the adult dose

c. Half of the adult dose Adult dosing schedules exceed the amount of drug necessary to produce a therapeutic response in a child, which places the child at risk for drug overdose. Pediatric dosing formulations reported by drug manufacturers are approximately one-half of the adult dose.

When should EMS be activated when treating an emergency involving an asthmatic client? a. At the beginning of the emergency b. After a bronchodilator and oxygen have been administered c. If the bronchodilator fails to stop the asthma attack d. As soon as CAB has been assessed

c. If the bronchodilator fails to stop the asthma attack If the bronchodilator fails to stop the asthma attack, EMS should be contacted. There is no need to contact EMS at the onset of an asthma attack; under normal circumstances, the bronchodilator should take care of the emergency. EMS will not need to be contacted for an asthma attack unless the bronchodilator is ineffective. Under normal conditions the bronchodilator will be effective in stopping the asthma attack; EMS will not need to be contacted.

All of the following are radiographic appearances in Sickle cell anemia EXCEPT one. Which is the EXCEPTION? a. Decrease in number of trabeculae in the jaws, with large marrow spaces b. Changes in the lamina dura c. Lamina dura not affected d. Perpendicular trabeculations radiating outward, giving a hair-on-end appearance in the skull

c. Lamina dura not affected

The amino acid tryptophan can be converted into which water-soluble vitamin? a. Pantothenic acid b. Thiamin c. Niacin d. Riboflavin

c. Niacin Niacin is found in protein foods such as fish, beef, turkey, and chicken. However, 1 mg of niacin can be converted from 60 mg of the amino acid tryptophan.

You have been monitoring your client's vital signs and assessing CAB, and he goes into cardiac arrest. You will have to use the defibrillator. Where will you put the electrodes? a. The opposite side of the pacemaker b. The defibrillator cannot be used on a client with a pace c. One inch to the side of the pacemaker d. Directly over the pacemaker

c. One inch to the side of the pacemaker Defibrillator pads should be placed at least one inch to the side of the pacemaker. Defibrillator pads cannot be placed directly over a pacemaker. Defibrillator pads can be placed on the same side as the pacemaker but must be at least one inch to the side of the pacemaker. If a defibrillator is necessary, it must be used to resuscitate the client. Pads placed in the correct position will not harm the pacemaker.

Which of the following is associated with the loss of lamina dura and a cotton-wool radiographic appearance of bone? a. Osteogeneis imperfecta b. Type I diabetes mellitus c. Paget's disease d. Fibrous dysplasia

c. Paget's disease Radiographic evidence reveals cotton-wool appearance and loss of lamina dura with Paget's disease. With type I diabetes mellitus, may show evidence of bone loss associated with periodontal disease; with fibrous dysplasia, may show a diffuse radiopacity. Osteogenesis imperfect presents with abnormally formed bones that fracture easily and abnormal dentin formation.

All of the following medications cause drug-influenced gingival enlargement as a side effect EXCEPT one. Which one is the EXCEPTION? a. Phenytoin (Dilantin) b. Cyclosporine (Sandimmune) c. Propranolol (Inderal) d. Nifedipine (Procardia)

c. Propranolol (Inderal) All of the medications listed are associated with causing gingival enlargement, except propranolol (Inderal) which is a nonselective beta blocker. As many as 60% of users of phenytoin (Dilantin) experience gingival enlargement, which may require surgical excision when severe. Good oral hygiene can limit the extent and severity of the lesion.

Which of the following is the most important factor for consideration when determining whether to proceed with dental hygiene treatment with a medicated client? a. Presence of environmental safety hazards b. Availability of supportive emergency equipment c. Risk for compromising the client's health d. Use of personal protective equipment

c. Risk for compromising the client's health The most important considerations to take into account when deciding whether or not to provide dental hygiene treatment are always the client's health and safety. If there is any doubt that the treatment provided will compromise the client's health and safety, then treatment

Which of the following emergencies occurs when the cells do not receive enough oxygen because blood is not circulating through the tissues adequately? a. Asthma b. Hypertension c. Shock d. Hyperventilation

c. Shock Shock is the result from the blood inadequately circulating through the tissue which prevents the cells from getting enough oxygen.

In which stage of gingivitis do rete pegs extend into the connective tissue? a. Stage I Gingivitis (Initial/Subclinical Stage) b. Stage II Gingivitis (Early Stage) c. Stage III Gingivitis (Established Stage) d. Stage IV Gingivitis (Advanced Stage)

c. Stage III Gingivitis (Established Stage)

Which of the following minerals is used to preserve foods? a. Chloride b. Phosphorus c. Sulfur d. Sodium

c. Sulfur Sulfur is present in all proteins and in the vitamins biotin and thiamin. Aids in maintaining a normal acid-base balance in the body. Sulfur compounds preserve foods.

Which of the following is an adverse drug event that results in permanent tissue damage? a. Hypersensitivity reaction b. Therapeutic effect c. Toxicity reaction d. Side effect

c. Toxicity reaction Drug toxicity is an adverse drug reaction whereby the drug or its metabolite induces cell damage and/or cell death. Toxicity reactions cause permanent damage at the cellular level; however this damage may ultimately affect organ function. Toxicity reactions are described according to the organ affected—for example, cardiotoxicity, hepatotoxicity, nephrotoxicity, ototoxicity, and neurotoxicity.

Which variable is used to determine pediatric drug dosage? a. Age b. Total body surface c. Weight d. Height

c. Weight Adult dosing schedules exceed the amount of drug necessary to produce a therapeutic response in a child, which places the child at risk for drug overdose. Because the mucous membranes and skin of a child are permeable, a drug can readily cross these surface areas and reach higher blood levels. Accurate dosing for pediatric clients is based on the weight of the child, typically based on a calculation of milligrams per kilogram of body weight.

Early CPR is an important link in the cardiac chain of survival because a. it prevents heart attacks. b. most cardiac arrest victims do not need defibrillation. c. it helps circulate blood that contains oxygen to the vital organs until an automated external defibrillator (AED) is ready to use or advanced medical personnel arrive. d. it helps restart the heart.

c. it helps circulate blood that contains oxygen to the vital organs until an automated external defibrillator (AED) is CPR helps supply oxygen to the brain and other vital organs to keep the victim alive until an AED is used or advanced medical care is given. Effective chest compressions circulate blood to the victim's brain and other vital organs. Chest compressions can also increase the likelihood that a successful shock can be delivered to a victim suffering a sudden cardiac arrest, especially if more than 4 minutes have elapsed since the victim's collapse.

ends with suffix - pine Ex: Nifedipine

calcium channel blocker

Bud stage

cells condense to develop into a tooth germ.

The radiographic error in the maxillary central incisor can be corrected by (case F)

centering the receptor evenly over the central incisors. (case F)

The process whereby phagocytic cells are attracted to the vicinity of pathogens is called

chemotaxis

Shingles is caused by the same virus that causes

chickenpox.

During intraoral examination of the occlusal surface of the mandibular first right premolar, a carious lesion with a mass of reddish tissue protruding from the large cavity is observed. The patient is a 35 year old female and has no symptoms. The mass of tissue is called

chronic hyperplastic pulpitis

According to G.V Black's classification of caries and restorations, which of the following describes the restoration on tooth #7? (case I)

class I (case I)

Radiographically tooth # 29 has (case C)

class II dental caries. (Case c)

The mandibular right first molar exhibits a furcation involvement MOST appropriately classified as (case H)

class III or IV. (Case H)

Water sticking to water demonstrates the principle of

cohesion

T-test

compares the means (averages) of two different studies

ANOVA (analysis of variance)

compares the means of three or more studies

Functions of the adult liver include all of the following EXCEPT one. Which one is the EXCEPTION?

concentration of bile.

Microscopic functional units that make up the bulk of the kidney are called

cortical nephrons

A drug that alters the pH of the stomach alters which aspect of pharmacokinetics of a second administered drug? a. Distribution b. Metabolism c. Excretion d. Absorption

d. Absorption The pH of the stomach affects drug dissociation, meaning how much of the drug dose is found as either a weak acid or a weak base. Weak acids are more readily absorbed from the stomach, which is a highly acidic environment, due to the presence of hydrochloric acid (low pH). The drugs that are weak bases are not absorbed as readily from the stomach. Furthermore, when the pH of the stomach is neutralized (e.g., with antacids), weak acids will no longer be absorbed as easily. Therefore the acidity or neutrality of the stomach can greatly affect whether or not a drug is absorbed into the circulation.

Which of the following is a function of vitamin E? a. Promotes the mineralization of teeth b. Aids in the prevention of night blindness c. Synthesizes prothrombin d. Acts as an antioxidant

d. Acts as an antioxidant Vitamin E is a fat-soluble antioxidant; protects other substances from oxidation. Also protects RBC membranes.

Which automated external defibrillator (AED) can be used on infants and children ages 1 to 8 years? a. Manual b. Adult automatic c. Pediatric-capable d. All choices

d. All choices All choices are correct; defibrillators can be used on infants and children ages 1 to 8 years. The manual defibrillator and pediatric-capable defibrillator are preferred.

Case B After dismissing his last patient of the day, Preston Miller, RDH, returns to his treatment room. He is in a rush because his daughter's softball team has a game tonight and he is the coach. Preston hurriedly closes his instrument cassette, not taking care to arrange the instruments beforehand. In the instrument-processing area, he lifts the cassette off the metal tray to place it into the ultrasonic unit. Unknowingly, an explorer tip is protruding from a hole in the cassette and penetrates his finger through the exam glove. He removes his gloves and sees a small amount of blood forming on his index finger. Additionally, Preston has received the HBV vaccine and has verified immunity to HBV. Preston, the employee, has sustained a percutaneous injury and may have been exposed to a bloodborne pathogen. In this situation, what is the responsibility of his employer according to OSHA? a. None; an employee works at his own risk b. Give the employee the following day off with pay c. Pay for the treatment of any acquired disease d. Arrange for a consultation with a qualified HCP

d. Arrange for a consultation with a qualified HCP

During his appointment, your client appears flushed, anxious, and short-tempered. He begins to cough and wheeze, is experiencing dyspnea, and begins to appear cyanotic. The emergency your client is experiencing is related to which of the following? a. Overdose reaction to local anesthetic b. Congestive heart failure c. Angina d. Asthma

d. Asthma Signs and symptoms of asthma include onset of a productive or a nonproductive cough, dyspnea (shortness of breath), tachypnea (increased respiratory rate), tachycardia (increased heart rate), anxiety and agitation, wheezing, and cyanosis, chest tightening, nostril flaring, and flushing.

Case B After dismissing his last patient of the day, Preston Miller, RDH, returns to his treatment room. He is in a rush because his daughter's softball team has a game tonight and he is the coach. Preston hurriedly closes his instrument cassette, not taking care to arrange the instruments beforehand. In the instrument-processing area, he lifts the cassette off the metal tray to place it into the ultrasonic unit. Unknowingly, an explorer tip is protruding from a hole in the cassette and penetrates his finger through the exam glove. He removes his gloves and sees a small amount of blood forming on his index finger. Additionally, Preston has received the HBV vaccine and has verified immunity to HBV. What is the source of microbes? a. Dental unit water b. Hygienist's hands c. Contaminated droplets d. Contaminated instrument

d. Contaminated instrument

Rickets is a disease associated with which vitamin deficiency? a. A b. E c. C d. D

d. D Rickets is a bone disease that affects children as a result of vitamin D deficiency. Bones soften because of failure to calcify normally; include bowed legs.

Clients who experience taste alteration as a medication-induced side effect often exhibit oral health behaviors that increase their risk for developing which condition? a. Periodontal disease b. Fungal infections c. Aphthous ulcers d. Dental caries

d. Dental caries When clients experience taste alteration from their medication use, they will often demonstrate excessive mouthwash use, suck on candies and mints, or chew gum in an effort to mask the aftertaste. If these clients choose products that contain sugar, these behaviors may contribute to caries formation, especially along the gingival margin. Clients should be encouraged to use xylitol-containing products.

Which of the following conditions occurs as a result of lowered bacterial plaque pH? a. Recession b. Aphthous ulcers c. Herpetic lesions d. Dentinal hypersensitivity

d. Dentinal hypersensitivity The loss of salivary buffers in a xerostomic client leads to a drop in intraoral pH. The acidity of the saliva that bathes the teeth, coupled with an increased bacterial plaque biofilm coating the teeth, contributes to dentinal hypersensitivity. Hypersensitivity may also occur as a result of the erosion of enamel and cementum by acids found in gastrointestinal fluids, food, and beverages.

Which of the following foods would be MOST cariogenic? a. Soda pop b. Milk c. Cake d. Dried apricots

d. Dried apricots Consistency of food plays a role in caries development. Food with sticky consistency will remain on the teeth longer and have slower clearance time. Dried apricots, like raisins, are dried fruit and remain on the teeth longer than other foods. Retention is a factor contributing to the initiation and progression of dental decay. Although soda pop, cake, and milk contain moderate to large amounts of sugar, they are cleared from the oral cavity MORE quickly.

What type of effect is seen in a person who experiences a response to a drug that is qualitatively different from the usual or expected response? a. Pathologic influence b. Drug toxicity c. Hypersensitivity reaction d. Drug idiosyncrasy

d. Drug idiosyncrasy Clients who experience a unique response to a drug that is qualitatively different from the desired therapeutic effect demonstrate an idiosyncratic response.

Which of the following causes irreversible injury that leads to cell death? a. Swelling of cytoplasm b. Brief hypoxia c. Condensation of cytoplasm d. Fragmentation of nucleus

d. Fragmentation of nucleus Damage to the nucleus of the cell and therefore to the genetic apparatus of the cell is considered irreversible cell injury.

Which of the following is considered endogenous cancer? a. Radiation b. Viruses c. Chemicals d. Genetic anomalies

d. Genetic anomalies Endogenous carcinogens include genetic factors, hormonal disturbances, immunological and nutritional factors, and aging.

Sorbitol, which is a sugar alcohol used in diabetic foods and sugarless gum, is made from a. Mannitol b. Fructose c. Galactose d. Glucose

d. Glucose Sorbitol is a sugar alcohol made from glucose through hydrogenation. Sugar alcohols are carbohydrates and yield approximately same energy as sucrose (4 kcal/g); both are nutritive sweeteners. Advantage that sugar alcohols have over sucrose is that they do not promote caries because they are not easily metabolized by bacteria in the mouth. Fructose is a monosaccharide and sweetest of all sugars; found in honey, fruits, and corn syrup. Mannitol is another sugar alcohol made from mannose and galactose.

Which of the following is the most common type of shock and can be caused by dehydration from vomiting and diarrhea? a. Distributive shock b. Cardiogenic shock c. Neurogenic shock d. Hypovolemic shock

d. Hypovolemic shock Hypovolemic shock is caused by severe hemorrhage or dehydration which can lead to inadequate blood volume or venous return.

Case B After dismissing his last patient of the day, Preston Miller, RDH, returns to his treatment room. He is in a rush because his daughter's softball team has a game tonight and he is the coach. Preston hurriedly closes his instrument cassette, not taking care to arrange the instruments beforehand. In the instrument-processing area, he lifts the cassette off the metal tray to place it into the ultrasonic unit. Unknowingly, an explorer tip is protruding from a hole in the cassette and penetrates his finger through the exam glove. He removes his gloves and sees a small amount of blood forming on his index finger. Additionally, Preston has received the HBV vaccine and has verified immunity to HBV. Which of the following modes of disease transmission is depicted in this case? a. Direct contact b. Droplet infection c. Airborne infection d. Indirect contact

d. Indirect contact

Wickham Striae and desquamative gingivitis are characteristic features of which one of the following; a. Erythema multiforme b. Cannon disease c. Pemphigus d. Lichen Planus

d. Lichen Planus Lichen Planus has the characteristic Wickham's Striae or lace-like pattern on the mucosa and desquamative gingivitis; Erythema multiforme is a hypersensitivity reaction. The onset is explosive and ulcerations on the mucosa and crusted lips are characteristic features; Cannon disease is the same as white sponge nevus, a hereditary condition. Clinically a white folded appearance to the buccal mucosa; Pemphigus has bullous or blister-type lesions. Although it can have desquamative gingivitis.

Primordial cysts are most often found radiographically: a. Around a supernumerary tooth b. In the anterior maxillary regions c. In the mandibular canine and first premolar areas d. Posterior to erupted third molars or in place of a tooth that was never present

d. Posterior to erupted third molars or in place of a tooth that was never present Primordial cysts are found posterior to erupted third molars or in the place of a tooth that was never present; The maxillary anterior region is not the area where primordial cysts are usually found; If associated with a supernumerary tooth, the cyst is usually a dentigerous cyst because a "tooth is present."; The mandibular canine and premolar regions are not common locations of a primordial cyst.

What circulatory system changes take place during inflammation? a. Decrease in blood vessel permeability b. Inhibition of diapedesis of leukocytes c. Constriction of arterioles d. Release of chemical mediators such as histamine

d. Release of chemical mediators such as histamine Pathogenesis of inflammation has circulatory and cellular changes; includes increases in blood flow and blood vessel permeability because of release of chemical mediators. Cellular changes include emigration (diapedesis) of leukocytes (WBC's) and phagocytosis.

All of the following characteristics apply to drug side effects EXCEPT one. Which one is the EXCEPTION? a. Reversible b. Dose related c. Tolerable d. Unpredictable

d. Unpredictable The Food and Drug Administration requires drug manufacturers to report all known adverse drug reactions, including side effects. Side effects are reported by body system and by the percentage of the population affected. Side effects are dose-related and are reversible on discontinuation of the drug. The vast majority of side effects are tolerable, meaning that they cause inconvenience or discomfort, but only rarely signify life-threatening events.

All the following viruses are bloodborne pathogens EXCEPT: a. Human immunodeficiency virus b. Hepatitis C virus c. Hepatitis B virus d. Varicella-zoster virus

d. Varicella-zoster virus

Maturation stage

dental tissues develop into mature tissues and undergo calcification.

Cell junctions that are attached by cell-to-cell mechanisms are called

desmosomes

The MOST common cause of tooth mobility is

destruction of the periodontium.

he MOST probable cause for the alignment of teeth #8 and #9 is (case C)

developmental. (case C)

To safely manage environmental and workplace safety issues related to disposal or storage of extracted teeth with amalgam restorations such teeth should be

disinfected with bleach.

Stratified sampling

divides groups into subgroups before a selection.

If a dental hygienist's license to practice dental hygiene is in jeopardy, the dental hygienist must first be given notice of the violation, a hearing, and an opportunity to respond before license revocation or suspension. This process exemplifies the principles of

due process.

Bell stage

enamel and dentin begin to form.

Cap stage

enamel organ, dental papilla, and dental sac are formed.

Apposition stage

enamel, dentin, and cementum are secreted.

The patient has 5 mm periodontal pocket depths (PPD) on teeth #2, #3, and #4. This measurement is taken from the (case I)

epithelial attachment to the gingival margin. (case I)

he most numerous type of red blood cell is/are the

erythrocytes.

The spacing observed between the anterior maxillary teeth is (case B)

excessive and indicates diastemas between the permanent teeth (case B)

The cause of the generalized white and discolored areas observed on most of the teeth is MOST likely caused by (case B)

fluorosis (case B)

The part of an instrument's shank that allows the working end to be adapted to the tooth surface is called the

functional shank.

Mushroom-shaped elevations scattered over the anterior third of the dorsum of the tongue are known as (case E)

fungiform papillae. (Case E)

A type of electrochemical corrosion that occurs to due contact of dissimilar metals is known as

galvanic corrosion

A localized purulent infection involving the marginal gingiva or interdental papilla that does not involve the underlying periodontium is called

gingival abscess

One indication for performing a gingivectomy is a(an)

gingival overgrowth

Random sampling

gives every member an equal chance to be selected.

Because _______ materials demonstrate true bonding between materials and dentin/enamel, they are widely used for Class V restorations.

glass ionomer

On x-ray examination, a radiolucent, pear-shaped lesion is found distal to a patient's maxillary right lateral incisor causing divergence of the lateral and canine roots. The lesion is likely a(an)

globulomaxillary cyst.

During endodontic therapy, the material used to seal the root canals of teeth #4, #13, and #19 is MOST likely (case A)

gutta percha. (case A)

After periodontal surgery, the embedding of new periodontal ligament fibers into new cementum and formation of a new gingival attachment in an area previously degenerated by disease is referred to as

healing by new attachment.

A university dental hygiene school received a state-funded grant to improve the oral health status of senior citizens in a rural area of the United States. The funding was for $50,000 over a 1-year period and covered expenses for salaries, equipment, supplies, and education. It was determined that high rates of root caries, root sensitivity, and gum disease were present in this population. The dental hygiene school decided to address these needs by implementing an oral health program in its clinic and to determine the effectiveness of two different types of desensitizing agents on treating root sensitivity.The local dental hygienists' association offered its services to provide educational sessions to the senior citizens. During these educational sessions, toothbrushes, dental floss, floss threaders, and denture brushes were made available for distribution. The distribution of dental products as part of a dental health program for senior citizens is indicative of

health promotion

The dark brownish coloration of gingival and alveolar mucosa in numerous areas is caused by

hereditary factors

A patient who tolerates pain well and shows little reaction to pain is said to have a

high pain threshold

A combination of antibody and antigen is a(n)

immune complex.

The pale appearance of this patient's radiographs is caused by (case G)

inadequate development time. (case G)

Local anesthetic solutions that contain a vasoconstrictor have all of the following properties EXCEPT one. Which one is the EXCEPTION

increase risk of systemic toxicity

Swelling of the parotid glands, an oral manifestation of chronic alcohol use disorder and dependence, most directly causes Prolonged alcohol use MOST commonly increases the incidence of damage to the (case C)

increased caries incidence. (case C)

The technique error in the mandibular right premolar periapical radiograph can be corrected by (case I)

increasing the KVP setting. (case I)

If a child accidentally ingests a toxic quantity of fluoride, the hygienist should FIRST

induce vomiting by administration of ipecac syrup

The BEST way to describe the patient's overall gingival margin is (case E)

inflamed (case E)

A state-hired full-time public health dentist and dental hygienist are responsible for collecting data on the dental plaque rates of elementary school-aged children. Correlational studies are planned to determine strength of data between genders and among geographical locations of the elementary schools. It was decided to screen children in grades K, 2, 4, and 6 within 25 schools throughout the state. The 25 schools were chosen on the basis of geographic locations and number of students enrolled. All students within each of the chosen grades would be part of the study. Three other public health dental hygienists were hired part time to help in the data collection process. When data collection was completed, analyzed, and interpreted, the public health dentist and the dental hygienist would decide on an oral health program to best meet the needs of the elementary school-aged children of the state.Calculation of the Oral Hygiene Index-Simplified (OHI-S) resulted in an overall mean score of 2.75 on a scale of 0 to 3. The MOST effective approach to improve the plaque scores of elementary students in this state is to

initiate a 2-year plaque control program in elementary schools.

A university dental hygiene school received a state-funded grant to improve the oral health status of senior citizens in a rural area of the United States. The funding was for $50,000 over a 1-year period and covered expenses for salaries, equipment, supplies, and education. It was determined that high rates of root caries, root sensitivity, and gum disease were present in this population. The dental hygiene school decided to address these needs by implementing an oral health program in its clinic and to determine the effectiveness of two different types of desensitizing agents on treating root sensitivity.Four dental hygiene clinical instructors will document the patient's response to each desensitizing agent. To ensure consistency of documentation among the four instructors, a calibration session is scheduled. This is an example of

inter-rater reliability

The radiopaque area on the mesial surface of tooth #2 in the right premolar bitewing is (case E)

interproximal subgingival calculus (case E).

Spine

is a blunt pointed projection, (e.g., anterior nasal spine in the midline of the maxilla).

Beri-beri

is a condition caused by vitamin B1 deficiency.

Marasmus

is a protein deficiency. Muscles are weakened but edema is not seen in this condition. Marasmus is most common in children of 6 to 18 months of age in impoverished nations.

Ridge

is a raised margin, (e.g., internal oblique ridge runs along the inside of the mandible).

Borax

is a retarder that increases setting time.

Tuberosity

is a rough prominence, usually attaches muscles and tendons, (e.g., maxillary tuberosity is connected to the meidial pterygoid muscle).

Ischemic stroke

is an obstruction within a blood vessel that is supplying blood to the brain. The obstruction can be either embolic or thrombotic.

Condyle

is an oval prominence and usually involves the joints, (e.g., condyle of the mandible forms the TMJ).

Kwashiorkor

is caused by a severe protein deficiency that begins at approximately 2 years of age. It is characterized by growth retardation, listlessness, edema in abdomen and legs (abdomen swelling because of liver enlargement).

When administering local anesthesia for this patient, use of a vasoconstrictor is (case H)

is contraindicated due to his blood pressure (140/90 mm Hg). (case H)

Primary prevention

is the most basic form of treatment focused on the prevention of a disease (e.g., prophylaxis, sealants, fluoride varnishes).

The alignment of teeth #24 and 25 in relation to the patient's occlusion are referred to as (case A)

labioversion/linguoversion. (case A)

The term for written defamation of character is

libel.

Habit:

lifestyle has changed.

Alcohol abuse is associated with each of the following oral conditions except

lipoma.

The byproducts from smoking are eliminated from the body by

liver metabolism

the most radiosensitive cell

lymphatic

Which of the following BEST describes the radiopaque areas superimposed on the root surfaces of teeth #20, #21, #22, #27, #28, and #29? (case I)

mandibular tori (case I)

Which tooth is known for its bifurcated root?

maxillary first premolar

The inverted Y in the maxillary left canine periapical film represents the intersection of the (case F)

maxillary sinus and nasal fossa. (case F)

P-value

measures result that occurred by chance during an experiment

Chi-square test

measures the difference between the observed and expected values, not means

A structure in the cell that contains a single, circular DNA, and synthesizes adenosine triphosphate (ATP) is the

mitochondria.

The technique error in the upper left molar periapical radiograph can be corrected by (case C)

moving the film or receptor more posteriorly (case C)

(Case J) The dark discoloration on the facial marginal gingiva of tooth #9 is most likely due to

non-vital status of the tooth. (case J)

Cardiac output is determined by the

number of heart beats per minute and stroke volume (SV).

Blood type is determined by the assortment of glycolipids and glycoproteins, called isoantigens, present on the surfaces of red blood cells (RBCs). Surfaces of type A red blood cells display

only antigen A.

Which of the following BEST describes the relationship of the incisal occlusion of anterior teeth? (case B)

overjet (case B)

The radiopaque line observed on the panoramic radiograph that runs horizontally across the maxilla from molar to molar is caused by the nose. (case A)

palate (case A)

The BEST rationale for the 5-mm probe reading on the mesiobuccal of tooth #18 is (case B)

partially erupted tooth (Case B)

Action:

patient acts to resolve the problem. New behaviors are formed.

Awareness:

patient knows about a problem but does not take actions.

Self-interest:

patient knows about the problem and shows interest in taking actions.

Unawareness:

patient lacks information and/or has incorrect information about a problem.

Involvement:

patient wants more knowledge and wants to participate.

Which of the following oral tissues receives stimuli from nociceptors, mechanoreceptors, and proprioceptors?

periodontium

Bacterial organisms stain differently on the basis of

permeability of their cell walls

What is the first action the dental hygienist should take if this patient describes that he has chest pain and labored breathing? (case I)

place pt in upright position (case I)

The junction of three surfaces on the crown of a tooth is referred to as a(n)

point angle

The liquid or matrix components used in both polycarboxylate and traditional glass ionomer cements is

polyacrylic acid in water.

When handling and disinfecting impressions, dentures, and appliances, the primary goal of infection control is to

prevent cross-contamination.

After this patient undergoes nonsurgical periodontal therapy (NSPT), the primary purpose of the maintenance phase of the dental hygiene care process is to (case E)

prevent recurrence of the disease. (case E)

A Mission of Mercy (M.O.M.) event was planned for an urban city in a southern state of the United States. This 2-day event was designed to meet the critical needs of underserved people of all ages by providing free dental care to as many adults and children as time, volunteers, and supplies would allow. Local dentists, dental hygienists, dental assistants, dental students, dental hygiene students, and dental assisting students volunteered for the event. Local politicians and two state representatives were also in attendance. Before receiving dental treatment, informed consent was required of all patients. A 10-question patient survey was distributed, and volunteers reviewed each question with patients while the patients were completing the questionnaire. Confidential questions included information on age, gender, education, family income, address, and past dental experiences. At the end of each patient's treatment and before each volunteer completed his or her shift, the patient was asked to complete an evaluation survey. There was an 88% response rate, and 79% answered that their experience was positive.The PRIMARY goal of this dental event was to

promote the need to seek future dental care.

People with anorexia nervosa are commonly deficient in

protein

Dental and dental hygiene practice acts are enacted to protect the

public.

Odontogenic myxoma, an odontogenic neoplasm that originates from mesenchymal tissue of tooth germ

radiographically appear as unilocular or multi-locular radiolucency; may have a "honeycomb" or "soap bubble" appearance.

A Mission of Mercy (M.O.M.) event was planned for an urban city in a southern state of the United States. This 2-day event was designed to meet the critical needs of underserved people of all ages by providing free dental care to as many adults and children as time, volunteers, and supplies would allow. Local dentists, dental hygienists, dental assistants, dental students, dental hygiene students, and dental assisting students volunteered for the event. Local politicians and two state representatives were also in attendance. Before receiving dental treatment, informed consent was required of all patients. A 10-question patient survey was distributed, and volunteers reviewed each question with patients while the patients were completing the questionnaire. Confidential questions included information on age, gender, education, family income, address, and past dental experiences. At the end of each patient's treatment and before each volunteer completed his or her shift, the patient was asked to complete an evaluation survey. There was an 88% response rate, and 79% answered that their experience was positive.For collection and analysis of research data, the measurement scale characterized by equal intervals along the scale that has presence of absolute zero and is utilized in most dental indices is the

ratio scale.

Reduction in probing depth following periodontal debridement is generally attributed to

reattachment of periodontal ligament fibers.

Agreement between states for recognition of dental hygiene licensure to practice in those respective states is referred to as

reciprocity.

Secondary prevention

refers the termination of a disease and the restoration of diseased oral tissues (e.g., amalgam placement, periodontal therapy).

Tertiary prevention

refers to the replacement of lost tissues (e.g., denture, implant).

Which of the following is NOT a "caries disease indicator" in the caries management by risk assessment approach (CAMBRA)? (case I)

restorations placed within the past 3 years as a result of caries (case I)

Systemic sampling

selects every nth member on a list.

Convenient sampling

selects subjects based on availability.

When mixing alginate material, addition of warm water will

shorten the gelation (setting) time.

During patient preparation for the panoramic radiograph, the thyroid collar (case A)

should be removed because it can block part of the beam (case A)

Normal blood pressure

should be under 120/80 mmHg

The term for verbal defamation of character is

slander.

This patient has class I mobility on tooth #2. The BEST description for this classification of mobility is (case D)

slight mobility with up to 1 mm of horizontal displacement in a facial-lingual direction. (case D)

The self-applied 1.1% (5000 parts per million) fluoride gel that can be applied as a brush-on or tray application contains

sodium fluoride (NaF).

At the end of oral prophylaxis, the dental hygienist applied fluoride varnish to the patient's teeth. The active ingredient in fluoride varnish is (case B)

sodium fluoride at 5% (case B)

According to the American Association of Periodontology, this patient's periodontal status in her maxillary right quadrant is classified as (case E)

stage III periodontitis. (case E)

When a restoration is improperly placed in a patient's mouth, the dentist is not meeting the profession's

standard of care

The primitive oral cavity is/are called the

stomodeum

The type of epithelial tissue that lines the oral cavity is called

stratified squamous epithelium

The lingual artery, an anterior branch of the external carotid artery supplies blood to each of the follow areas EXCEPT one. Which one is the EXCEPTION?

sublingual salivary gland.

The bilateral radiolucent area in the molar region inferior to the mylohyoid ridge is known as the (case D)

submandibular fossa.(case D)

When reviewing a dentigerous cyst on a radiograph, the radiolucent area will

surround the crown of an unerupted or impacted tooth.

A chemical, biologic, or physical agent that causes birth defects is known as a

teratogen

Tooth development occurs in six stages:

the initiation stage, bud stage, cap stage, bell stage, apposition stage and maturation stage.

The paired cavities of bone visible as bilateral radiolucencies that originate at the maxillary canine region and extend posteriorly is/are (case C)

the maxillary sinus cavities. (case c)

Initiation stage

the oral epithelium grows deeper into the layer below the epithelium, called the mesenchyme, and produces the dental lamina, which is the band of earliest tooth structure.

Radiographic images will be blurred by (case B)

the patient moving. (case B)

Formerly known as biologic width, supracrestal attached tissue is

the space on the tooth surface occupied by the junctional epithelium and the connective tissue fibers.

Each of the following is an accurate statement regarding indirect digital radiographic imaging EXCEPT one. Which one is the EXCEPTION?

the traditional x-ray machine is no longer required

The type of material used for athletic mouthguards is referred to as (case B)

thermoplastic co-polymer. (case B)

The gingival recession noted on tooth #3 is MOST likely caused by (case A)

toothbrushing, smoking and attrition. (case A)

The redness on the marginal gingiva of tooth #11 is MOST likely caused by (case A)

unknown etiology (case A)

A university dental hygiene school received a state-funded grant to improve the oral health status of senior citizens in a rural area of the United States. The funding was for $50,000 over a 1-year period and covered expenses for salaries, equipment, supplies, and education. It was determined that high rates of root caries, root sensitivity, and gum disease were present in this population. The dental hygiene school decided to address these needs by implementing an oral health program in its clinic and to determine the effectiveness of two different types of desensitizing agents on treating root sensitivity.The data collection instrument selected to measure the effect of the desensitizing agent on root sensitivity has been used successfully in numerous studies with published results appearing in the Journal of Dental Hygiene. This indicates that the instrument is

valid.

A state-hired full-time public health dentist and dental hygienist are responsible for collecting data on the dental plaque rates of elementary school-aged children. Correlational studies are planned to determine strength of data between genders and among geographical locations of the elementary schools. It was decided to screen children in grades K, 2, 4, and 6 within 25 schools throughout the state. The 25 schools were chosen on the basis of geographic locations and number of students enrolled. All students within each of the chosen grades would be part of the study. Three other public health dental hygienists were hired part time to help in the data collection process. When data collection was completed, analyzed, and interpreted, the public health dentist and the dental hygienist would decide on an oral health program to best meet the needs of the elementary school-aged children of the state.After calculating the results of the correlation test between geographic location and plaque scores, the correlation coefficient (r-value) was 0.26. This relationship between these two variables is

weak.

The attached gingiva is generally

widest in the maxillary facial anterior areas and narrowest in the mandibular facial premolar areas

If a negligent act presents a scope-of-practice issue, a dental hygienist's actions should be

within the scope of practice for the particular jurisdiction

Vitamin C

(ascorbic acid) is associated with protein collagen formation, wound healing, and stronger immune system.

Vitamin D

(calciferol) helps in the absorption of calcium and phosphorus. It promotes mineralization of teeth and bones.

Vitamin A

(retinol) is important for night vision: healthy, oral mucosa, and sulcus (assists in the formation of ameloblasts and odontoblasts). Sources of vitamin A include carrots and squash.

Vitamin E

(tocopherol) acts as an antioxidant (prevents oxidation of vitamin).

With amelogenesis imperfecta, the enamel may have

- Abnormally small teeth - Yellow or brown discoloration - Pitting - Sensitivity - Open bite - Prone to breaking

Which of the following BEST describes this patient's vital signs? (case G)

. Blood pressure rate is considered higher than normal (case G)

Introduction of solid foods too early is the MOST likely cause of early childhood caries.Early tooth eruption is strongly correlated with early childhood caries (ECC).

. Both statements are FALSE

Which of the following is the primary cell involved in an acute inflammatory reaction?

. Neutrophil

transdermal systems for use in nicotine reduction systems includ? each of the following EXCEPT one. Which one is the EXCEPTION

. Nicorette

Which of the following medications prescribed to this patient is BEST used to reduce formation of blood clots? (case I)

. Plavix (case I)

Pellicle formation involves

. adhesion of salivary proteins to the tooth surface

The knowledge, skills and ability to provide oral health care effectively to people who are culturally different from oneself is known as

. cultural competence.

On the lingual roots of tooth #23 and tooth #24, a gingival cleft is present, and the marginal alveolar bone is denuded, forming a defect extending apical to the normal level and exposing an abnormal amount of root surface. This is characteristic of (case F)

. dehiscence (dehescience)

The BEST reason for the presence of this patient's maxillary diastema between teeth # 8 and #9 is (case J)

. frenal attachment and periodontitis(case J)

The 15 mg of lansoprazole (Prevacid) taken by this patient is indicated for (case B)

. heartburn. (case B)

When treating a patient on an implant maintenance regimen, it is crucial that the dental hygienist select

. plastic and/or titanium instruments.

Sweetened and acidic beverages should be consumed

. with meals

Over the counter toothpaste usually contains

0.24% sodium fluoride

If the family of a 5-year-old child uses well water with a 0.4ppm concentration of fluoride, what dosage of fluoride supplement should the clinician prescribe for the child?

0.25 mg

The patient's mother reports that they live in the city and that the community water supply is fluoridated. What is the optimal amount of fluoride appropriate for the city water supply (case G)

0.7 ppm (case G)

The patient's mucogingival junction is located 3 mm from the center of the gingival margin on the facial surface of tooth #22. On the basis of the pocket measurement at this location, what amount of attached gingiva is present at the center of the facial surface of tooth #22? (case F)

1 mm (case F)

Case A Isabelle Sanchez, a registered dental hygienist (RDH), has been working in the same practice, substituting for a hygienist who is on maternity leave. Today, when she reports to work, Isabelle is told by the dental assistant that the elderly man whom Isabelle treated last Monday is in the hospital and has been diagnosed with Legionnaires' disease. During the last week, he had only left his house to go to his dental visit, and the water tested in his house was negative for Legionella. In hindsight, Isabelle realizes that in her treatment room, she did not see a water bottle attached to the master control unit that would have indicated the use of an independent water system. She just assumed that the dental unit water lines were being treated with another method. QUESTIONS: 1. What is the pathway of disease transmission? 2. Which mode of disease transmission is depicted? 3. What is the source of microbes? 4. Which infection control procedure would have prevented disease transmission?

1. Community to patient 2. Airborne infection 3. Dental unit water 4. Treating dental unit water lines

Under the AAP Classification of Periodontal and Peri-implant Diseases and Conditions (2017), which conditions are listed under "Periodontal Health and Gingival Diseases and Conditions?"

1. Gingivitis — Dental Biofilm Induced 2. Gingival Diseases — Non-dental biofilm induced

Under the AAP Classification of Periodontal and Peri-implant Diseases and Conditions (2017), which conditions are listed under "Periodontitis?"

1. Necrotizing Periodontal Diseases 2. Periodontitis as a Manifestation of Systemic Conditions 3. Periodontitis

Under the AAP Classification of Periodontal and Peri-implant Diseases and Conditions (2017), which conditions are listed under "Peri-implant Diseases and Conditions?"

1. Peri-Implant Health 2. Peri-Implant Mucositis 3. Peri-Implantitis 4. Peri-Implant Soft and Hard Tissue Deficiencies

Under the AAP Classification of Periodontal and Peri-implant Diseases and Conditions (2017), which conditions are listed under "Other Conditions Affecting the Periodontium?"

1. Systemic Diseases or Conditions Affecting the Periodontium 2. Periodontal Abscesses and Endodontic-Periodontal Lesions 3. Mucogingival Deformities and Conditions 4. Traumatic Occlusal Forces 5. Tooth and Prosthesis Related Factors

The clinician changed the 8 inches PID to a 16 inches PID. How will the beam intensity change?

1/4 times stronger

What is the normal Prothrombin time (PT)?

11-16 seconds

To determine if the client has calculus deposits on the proximal surfaces of the posterior teeth, which instrument with which stroke should be used?

11/12 explorer, fluid stroke

Pre-hypertension

120-129 systolic and less than 80 mmHg

Hypertension stage 1

130-139 or 80-89 diastolic mmHg

How many facial bones are there?

14 "My Mandible Chews Nine Very Large Zucchini Pizzas" Maxillae (2) Mandible Conchae (2) Nasal (2) Vomer Lacrimal (2) Zygomatic (2) Palatine (2)

Hypertension stage 2

140+ systolic or 90+ diastolic

The dental hygienist has started to debride the lingual aspect of teeth #24 and #25, using a sickle scaler to remove the supragingival calculus. The number of cutting edges per working end of a sickle scaler is _____ . The face and the lateral surface will meet in an angle of ______. (case E)

2, 70 to 80 degrees (case E)

A setting of 10 milliamperes (mA) with an exposure time of 3 seconds results in 30 mAs (milliamperes per second) of radiation exposure. To maintain the same density of the exposed radiograph when the mA is increased to 15, the time of exposure must be reduced to

2.0 seconds.

Dental treatment should be scheduled within what time period of kidney dialysis treatment.(Case C)

24 hours after dialysis (case C)

What is the most common preventive antibiotic regimen used for adults who do not have penicillin allergy?

2g Amoxicillin 1 hour prior to appointment

How many bones are there in the middle ear?

3 Malleus Incus Stapes

Waterlines of the dental unit should be flushed in between patients for

30 seconds

During the assessment phase of care, the dental hygienist determined that some of the patient's premolar teeth could benefit from application of for pit-and-fissure sealants. What percentage of phosphoric acid is contained in the various acid conditioning agents available for use during sealant placement? (case G)

30% to 50% (case G)

Etching technique for the placement of sealants involves

30-50% phosphoric acid

How many appointments are indicated for nonsurgical periodontal therapy (NSPT), and what are their intervals?

4 appointments: Every 6 weeks for re-evaluation

What is the amount of energy produced by 1 gram of protein?

4 kcal

Proteins and carbohydrates produce

4 kcal/g of energy

What is the critical pH for enamel demineralization?

4.5-5.5

What is the total limit for occupational exposure to radiation per year?

5 rem

What percentage inorganic material is present in cementum?

50%

Alcohol produces

7 kcal/g of energy

Probe depth is 3mm, the recession is 4mm. What is the CAL?

7 mm

What percentage inorganic material is present in dentin?

70%

How many cranial bones are there?

8 Frontal Parietal (2) Occipital Temporal (2) Sphenoid Ethmoid

Fats produce

9 kcal/g of energy

Six millimeters of recession is observed on tooth #3. Which of the following accurately reflects the clinical attachment level? (case A)

9 mm (case A)

Urine is composed of what percentage of water?

95%

What percentage inorganic material is present in enamel?

96%

You have been employed as a public health dental hygienist in a local health department to provide educational presentations for the participants in the WIC non-fluoridated program in a non-fluoridated community. The natural fluoride (f) concentration of the community water supply is 0.3ppm f. one of your first assignments is to present an educational program on basic oral health practices for culturally diverse pregnant teens in three alternative high schools that have daycare facilities. How much additional fluoride would be required to bring the fluoride concentration of the water supply to the optimal level? A. 0.4 ppm F B. 0.7 ppm F C. 1 ppm F D. 0.4 to 0.9 ppm

A. 0.4 ppm F

Approximately what concentration of fluoride do most commercially available dentifrices have? A. 1,000 ppm B. 100,000 ppm C. 10% D. 4%

A. 1,000 ppm

All of the following are key elements of obtaining informed refusal except one. Which one is the exception? A. An oral refusal is considered a legal agreement B. Explanation and justification for the recommended treatment and alternatives C. Documentation as to why the client is refusing treatment D. The risks of refusing treatment E. Signature of client, provider, and witness signatures

A. An oral refusal is considered a legal agreement

After a discussion with a client concerning proposed dental hygiene treatment, including alternatives, risks and benefits, and outcomes, the client accepts and signs an informed consent form. Which of the ethical principles guiding oral health care did the process of obtaining informed consent satisfy? A. Autonomy B. Beneficence C. Fair and equitable care D. Reliability or trustworthiness does not apply to the process of informed consent E. Truthfulness

A. Autonomy

A sealant is placed on permanent molars to help prevent which of the following? A. Class I caries B. Class II caries C. Class III caries D. Class IV caries E. Class V caries

A. Class I caries

Which of the following oral features are frequently associated with down syndrome? A. Deeply fissured tongue, mouth breathing, thickened lips B. Angle's class II occlusion, macrodontia, narrow palate C. Gingivitis, supernumerary teeth, macroglossia D. Atrophic lips, drooling, linear gingival erythema

A. Deeply fissured tongue, mouth breathing, thickened lips

Which of the following yields the most energy per gram? A. Fats/lipids B. Proteins C. Carbohydrates D. Alcohol

A. Fats/lipids

What change in alveolar bone on a dental radiograph indicates the early loss of bone? A. Fuzziness in the crest of the bone B. Faint cup-shaped areas interproximally C. Vertical or angular defects D. Furcation involvement

A. Fuzziness in the crest of the bone

After completing the dental hygiene assessment, the dental hygienist determines that Sandra, 14 years old, will require instruction over a series of appointments to improve her flossing technique. what should the dental hygienist evaluate to determine Sandra's progress? A. Health of interproximal gingiva B. Amount of dental plaque in interdental areas C. Sandra's flossing technique D. Radiographic appearance of interproximal areas

A. Health of interproximal gingiva

Each of the following describes generalized tetracycline stains except one. Which one is the exception? A. Initiated in adolescence B. Acquired in early childhood C. Considered intrinsic endogenous D. Acquired through fetal circulation E. Can be shades of gray, green, and yellow

A. Initiated in adolescence

From among the following choices, which of the following provides general sensation to the anterior two-thirds of the tongue? A. Lingual nerve B. Hypoglossal nerve C. Chorda Tympani nerve D. Glossopharyngeal nerve E. Lesser petrosal nerve

A. Lingual nerve

All of the following are steps indicated when a curette tip breaks during periodontal debridement except one. Which one is this exception? A. Placing the patient in a supine position B. Examining the mucobuccal fold and floor of the mouth C. Taking a radiograph to locate the tip D. Gently instrumenting the pocket to locate the tip

A. Placing the patient in a supine position

In monitoring the use of N2O, the patient should be in which stage of general anesthesia? A. Stage I B. Stage II C. Stage III D. Stage IV

A. Stage I

Which of the following instruments would be the best choice to initiate the removal of moderate calculus deposits? A. Standard ultrasonic tip B. Ultrasonic slim-diameter tip with a straight working end C. Set of rigid Gracey curettes 1/2, 11/12, and 13/14 D. Quétin furcation curet

A. Standard ultrasonic tip

Which treatment time is required for full mouth disinfection? A. Two separate 4-hour appointments, completed within 24 hours B. Four 60-minute appointments, completed at two-week intervals C. A single 1-hour appointment, with an optional reevaluation appointment, if bleeding occurs D. Six 90-minute appointments, plus an additional re-evaluation appointment for surgery

A. Two separate 4-hour appointments, completed within 24 hours

The microorganism most often associated with aggressive periodontitis is

A. actinomycetemocomitan

A dental hygienist is creating an oral health care program for participants at a local women, infants, and children program (WIC). The hygienist will develop a community profile to determine the participant's dental knowledge. instruction will include tooth brushing, oral physiotherapy techniques, and the relationship between diet and dental decay. The program will be offered for one month on days participants pick up food vouchers. by offering instruction at the WIC site, the dental hygienist hopes to educate caregivers and children early enough to prevent dental decay. Which provides information to parents on child developmental stages so the child's oral health needs can be adequately met? The statement "The oral health care program will improve the dental health of women and their children" is an example of the program A. goal B. Objective C. Evaluation D. intervention

A. goal

When a patient is experiencing a seizure in the dental chair, the clinician should A. move objects out of the way B. restrain the patient C. Expose the patient to plenty of light D. raise the chair to an upright position

A. move objects out of the way

When removing subgingival calculus deposits from the proximal surfaces of the anterior teeth which of the following instruments is ideal? A.) Area specific curette B.) Explorer C.) Universal curette D.) Sickle scaler

A.) Area specific curette

Which part of an atom is NOT responsible for its mass? A.) Electrons B.) Protons C.) Neutrons D.) Nucleus

A.) Electrons

The oral signs of diabetes include all of the following EXCEPT A.) Fast wound healing B.) Periodontal disease C.) Xerostomia D.) Oral candidiasis

A.) Fast wound healing

Which of the following is absorbed into the bloodstream the fastest? A.) Glucose B.) Lactose C.) Maltose D.) Sucrose

A.) Glucose

Which type of food is MORE cariogenic? A.) Gummy bears B.) Bread C.) Soda D.) Water

A.) Gummy bears

Which of the following is a diuretic? A.) Hydorcholorothiazide B.) Phenytoin C.) Losartan D.) Lisinopril

A.) Hydrochlorothiazide

Which of the following is a highly radiosensitive cell? A.) immature cell B.) Highly specialized cell C.) Mature cell D.) Slowly dividing cell

A.) Immature cell

Which one of the following is a negative reproduction? A.) Impression B.) Replica C.) Stone model

A.) Impression

Which symptom is related to Alzheimer's disease? A.) Inability to recall recent events B.) Resting tremors C.) Unilateral weakness D.) Bradykinesia

A.) Inability to recall recent events

In order to increase sharpness on a radiographic image, it is preferable to A.) Increase tube to object distance B.) Increase the size of the focal spot C.) Increase object to film distance D.) Use film with large crystals

A.) Increase tube to object distance

Which of the following is NOT correct regarding major aphthous ulcers? A.) Lesions appear as tiny vesicles B.) Lesions are usually painful C.) Lesions may take several weeks to heal D.) Lesions are commonly associated with immunosuppressive diseases

A.) Lesions appear as tiny vesicles

All of the following may result in radiographic images that are too light EXCEPT A.) Light exposure B.) Cold developer solution C.) Longer PID D.) Decreased kVp

A.) Light exposure

Which type of white blood cells are more commonly found in acute inflammation? A.) Neutrophils B.) Macrophages C.) Monocytes D.) Mast cells

A.) Neutrophils

When comparing primary and permanent molars, which of the following statements is TRUE? A.) Primary molars have larger pulp chambers B.) Primary molars have longer root trunks C.) Primary molars have thicker enamel layers D.) All of the above

A.) Primary molars have larger pulp chambers

Which repair process describes healing with minimal scarring? A.) Repair by primary intention B.) Repair by secondary intention C.) Repair by tertiary intention D.) Keloid

A.) Repair by primary intention

Which of the following describes a negative correlation? A.) The more the patient brushes his teeth, the lower the risk for caries B.) The more the patient eats candies, the more the risk for caries C.) The patients brushing time and caries risk are not related

A.) The more the patient brushes his teeth, the lower the risk for caries

Which of the following is NOT correct regarding furcations? A.) They can be detected on anterior teeth B.) They can be detected on the posterior teeth C.) They can be visible when recession is present D.) They can present a challenge to the clinician during instrumentation

A.) They can be detected on anterior teeth

Stimulant drugs such as Concerta, Ritalin, and Dexedrine are commonly used in the treatment of

ADHD

According to the American Society of Anesthesiologists classification, this patient's physical status can be classified as (Case H)

ASA III (Case H)

According to the American Society of Anesthesiologists classification, this patient's physical status can be classified as (case H)

ASA III. (case H)

Which of the following BEST describes the conditions on the cervical areas of mandibular premolar and canine teeth visible in the photographs? (case D)

Abfraction (case D)

The force of attraction between unlike atoms and molecules on two different surfaces is

Adhesion

During the assessment phase, the patient mentions that periodontal probing hurts and asks the dental hygienist to stop. Which of the following procedures is BEST to follow? (case H)

Administer local anesthesia per quadrant/sextant, then probe and debride that area (case H)

All of the following are appropriate treatment modifications for an individual with chronic obstructive pulmonary disease (COPD) EXCEPT one. Which one is the EXCEPTION?

Administering oxygen

Which type of bacteria cannot survive without oxygen?

Aerobic

Gram-positive rods appear how many days after plaque accumulation?

After 2-4 days

The pulp chambers of this patient's maxillary molars are slightly obliterated. What is the MOST likely cause for this occurrence?(case D)

Aging process (case D)

What is the mode of transmission for Mycobacterium tuberculosis?

Airborne

Which of the following conditions are present on the dorsal surface of this patient's tongue? (case J)

All of the above (case J)

A teenage patient just received a carpule of lidocaine. She suddenly feels itchy, her breathing becomes heavy, and her pulse is weaker. The clinician quickly checks the medical history and does not find any specific conditions that could cause such changes in the patient. What could the clinician suspect?

Allergic reaction

Which fibers of the principal fiber group extend from the cervical cementum and insert themselves into the alveolar crest?

Alveolar crest

All of the following are contraindicated for cosmetic polishing/selective polishing EXCEPT one. Which one is the EXCEPTION?

Amalgam

It is required by law that health care facilities contain a barrier-free design to accommodate patients who have a physical disability. By which of the following laws that ensure the rights of people with disabilities is this patient who is wheelchair bound most protected? (case F)

American with Disabilities Act (case F)

Which of the following aspects of metabolism includes de novo synthesis and requires DNA, RNA, mRNA, and rRNA?

Anabolism

Diagnosis in a dental office can be compared to which process in community oral health planning?

Analysis

A state-hired full-time public health dentist and dental hygienist are responsible for collecting data on the dental plaque rates of elementary school-aged children. Correlational studies are planned to determine strength of data between genders and among geographical locations of the elementary schools. It was decided to screen children in grades K, 2, 4, and 6 within 25 schools throughout the state. The 25 schools were chosen on the basis of geographic locations and number of students enrolled. All students within each of the chosen grades would be part of the study. Three other public health dental hygienists were hired part time to help in the data collection process. When data collection was completed, analyzed, and interpreted, the public health dentist and the dental hygienist would decide on an oral health program to best meet the needs of the elementary school-aged children of the state.Mean scores of the correlations between the following factors were established: geographic location and plaque rates gender and plaque rates geographic location and gender Of the following statistical analytic techniques (methods), which is the most appropriate selection?

Analysis of variance (ANOVA)

The medical term used for chest pain or chest discomfort is

Angina pectoris

ends with - tan Ex: Losartan

Angiotensin II receptor blockers (ARB)

ends with - pril Ex: Enalpril

Angiotensin converting enzyme (ACE) inhibitor

All of the following are adverse reactions associated with oral contraceptives EXCEPT one. Which one is the EXCEPTION?

Anorexia

Voluntary starvation related to distorted body images are characteristics of a person with

Anorexia nervosa

To make air-powder polishing MORE effective and comfortable, the angle of the tip to the tooth should be

Anterior teeth 60 degrees and posterior teeth 80 degrees

In a panoramic radiograph which of the following is the result a focal trough positioning error?

Anterior teeth appear blurred and magnified in size ANS

During hand-activated instrumentation (scaling), which type of immunity may be immediately conferred to a dental hygienist following an accidental curet laceration of the finger accident?

Artificial passive immunity

Hydrogen peroxide has been used in the oral cavity for a variety of purposes. These include all of the following EXCEPT one. Which one is the EXCEPTION?

As an essential oil

The patient says he is concerned about gagging while the dental hygienist takes alginate impressions. Which of the following will help reduce the patient's gagging sensation? (case G)

Asking the patient to concentrate on breathing through his nose (case G)

Which of the following is NOT recommended for a patient with asthma?

Aspirin

All of the following are recommended as successful individual-provider interactions to address tobacco cessation with patients EXCEPT one. Which one is the EXCEPTION?

Assist

During treatment of a patient with compromised pulmonary function, all of the following must be done EXCEPT one. Which one is the EXCEPTION?

Assure that the patient has been pre-medicated

Common themes to a code of ethics include each of the following EXCEPT one. Which one is the exception?

Assuring fairness in financial arrangements

All of the following are considered common upper respiratory infections EXCEPT one. Which one is the EXCEPTION?

Asthma

All of the following are risk factors of a cerebrovascular accident (stroke) EXCEPT one. Which one is the EXCEPTION?

Asthma

All of the following statements are true about atorvastatin calcium (Lipitor) medication EXCEPT one. Which one is the EXCEPTION?

Atorvastatin calcium is a medication used to treat arthritis

Bruxism is associated with which of the following defects that form within tooth structure?

Attrition

In which type of immunopathologic disease are the cells of the body no longer tolerated and the immune system treats them as antigens?

Autoimmune diseases

According to the learning ladder model, if the patient is aware of his periodontal disease, but is not taking actions to resolve the problem, his is showing?

Awareness

When measuring the cal on a patient whose gingival margin covers the CEJ, if the probe depth is 7 and the distance from the CEJ to the gm is 2, what is the cal? A. 2mm B. 5 mm C. 6 mm D. 9 mm

B. 5 mm

A tooth that is mobile >1mm in BL or MD direction (but less than 2mm) without vertical mobility is A. Class I B. Class II C. Class III D. None

B. Class II

Which of the following is NOT true about hand hygiene? A. Plain soap and water will kill bacteria. B. Gloves can be substituted for hand-washing. C. Alcohol-based hand rubs can be used as long as the hands are clean of debris. D. Long nails can harbor more bacteria than short nails.

B. Gloves can be substituted for hand washing

According to the classification system of the American Academy of Periodontology, which classification of periodontal disease is generalized aggressive periodontitis? A. II B. III C. IV D. V

B. III

Which immunoglobulin (antibody) predominates in a type I hypersensitivity reaction? A. IgA B. IgE C. IgG D. IgM

B. IgE

The most common location of oral cancer is the : A. Floor of the mouth B. Lateral border of the tongue C. Lip D. Pharynx

B. Lateral border of the tongue

A patient with an IQ of 40 has what level of intellectual disorder? A. Mild B. Moderate C. Severe D. Profound

B. Moderate

The Hippocratic oath is an example of an early ethical code that advocates health providers should do no harm. which universal ethical principle best supports this recommendation? A. Autonomy B. Non-maleficence C. Justice D. Beneficence E. Veracity

B. Non-maleficence

A dental hygienist is creating an oral health care program for participants at a local women, infants, and children program (WIC). The hygienist will develop a community profile to determine the participant's dental knowledge. instruction will include tooth brushing, oral physiotherapy techniques, and the relationship between diet and dental decay. The program will be offered for one month on days participants pick up food vouchers. by offering instruction at the WIC site, the dental hygienist hopes to educate caregivers and children early enough to prevent dental decay. Which level of prevention does the oral health care program address? A. Tertiary B. Primary C. Secondary D. Quaternary

B. Primary

Infective endocarditis prophylaxis with an antiinfective agent is indicated for patients who have had A. Mitral valve prolapse B. Prosthetic cardiac valve C. Rheumatic heart disease D. Atrial septal defect

B. Prosthetic cardiac valve

A dental hygienist is root planing and scaling and removes a portion of a client's amalgam during the procedure. The dental hygienist fails to inform the client, who later experiences pain and swallows a portion of the remaining restoration. The intentional tort committed is the following. A. Malpractice B. Standard negligence (Misrepresentation) C. Professional negligence D. Breach of contract

B. Standard negligence (Misrepresentation)

Which of the following is secondary prevention? A. When disease does not occur B. The treatment of early disease C. The progression of disease D. True prevention

B. The treatment of early disease

Pain associated with hypersensitivity can be described as the following. A. Constant pain B. Transient pain C. Intermittent pain D. Severe pain

B. Transient pain

Which of the following makes Behcet's syndrome different from recurrent ulcerative stomatitis? A. Bull's eye skin lesions B. Triad of locations of lesions (oral, eye, genital) C. Exudate from lesions D. Mesenchymal proliferations

B. Triad of locations of lesions (oral, eye, genital)

Using biologic indicators to test sterilization equipment, biologic monitors should be used once every A. Day B. Week C. Month D. Quarter

B. Week

The two earliest signs of gingival inflammation preceding established gingivitis are A. increased collagen fibers and vascularity B. increased gingival crevicular fluid production rate and bleeding from the gingival sulcus upon probing C. Gingival recession and increased bleeding on probing the sulcus D. Presence of calculus and bone loss E. Gnawing discomfort and ulceration of the marginal gingiva

B. increased gingival crevicular fluid production rate and bleeding from the gingival sulcus upon probing

Which of the following indicates the effectiveness of sterilization? A. Change of color on autoclave tape or bag B. live spore test C. Following time and temperature requirements exactly D. Monitoring patients for disease

B. live spore test

The following are the advice given to patients undergoing cancer therapy EXCEPT A.) Keep hydrating B.) Brush immediately after vomiting C.) Use sodium bicarbonate to raise the pH after vomiting D.) Avoid spicy, salty, and hot food

B.) Brush immediately after vomiting

Which antihypertensive agent is associated with gingival enlargement? A.) Atenolol B.) Nifedipine C.) Enalapril D.) Losartan

B.) Nifedipine

Which of the followings is NOT true regarding the geographic tongue? A.) It is categorized by erythematous patches surrounded by yellow borders B.) The affected area can have a rough texture C.) Patches can disappear and reappear in different locations D.) No treatment is necessary

B.) The affected area can have a rough texture

During the intraoral assessment, the dental hygienist observes that this patient does not have a midline shift although he is missing tooth #6. What is the rationale for the normal alignment of the anterior teeth? (case I)

Balance of the arch was possible due to missing tooth #13 (case I)

Before a patient agrees to a root canal therapy, the dentist explains that the treatment is urgent and proceeds with local anesthesia administration. In legal terms, the clinician is performing

Battery

A dental hygienist whose ethical principles include taking only necessary radiographs and maintaining equipment to prevent patient injury, such as replacing worn instruments, is demonstrating what type of ethical principle?

Beneficence

All of the following are true in regards to this patient's antihypertensive medications EXCEPT one. Which one is the EXCEPTION? (case H)

Beta-blockers are not effective in treating hypertension. (case H)

Incisors are used for

Biting and cutting

Which of the patient's vital signs needs further assessment? (case D)

Blood pressure (case D)

Which of the following conditions in this patient needs further assessment? (Case E)

Blood pressure (case E)

Which of the following BEST describes this patient's vital signs? (case B)

Blood pressure is considered high (case B)

Which of the following statements correctly describes the vital signs taken on a 25 year old patient? Blood pressure: 125/78 Pulse: 86 beats per minute Respiration: 13 respirations per minute

Blood pressure should be classified as elevated

Which of the following BEST describes the contour of the interdental papilla around teeth #23, #24, #25, and #26? (case A)

Blunted (case A)

A state-hired full-time public health dentist and dental hygienist are responsible for collecting data on the dental plaque rates of elementary school-aged children. Correlational studies are planned to determine strength of data between genders and among geographical locations of the elementary schools. It was decided to screen children in grades K, 2, 4, and 6 within 25 schools throughout the state. The 25 schools were chosen on the basis of geographic locations and number of students enrolled. All students within each of the chosen grades would be part of the study. Three other public health dental hygienists were hired part time to help in the data collection process. When data collection was completed, analyzed, and interpreted, the public health dentist and the dental hygienist would decide on an oral health program to best meet the needs of the elementary school-aged children of the state.

Board of education members, the school principals, and the parents

Edentulous patients will lose their vertical facial dimension over time. Which of the following can explain the reason for this change?

Bone resorption occurs when teeth are not present

Enamel maturation begins once enamel formation is complete.The hydroxyapatite crystals in enamel are one third the size of those in bone, dentin, and cementum. (case G)

Both statements are FALSE (case G)

A disadvantage of acidulated phosphate fluoride (APF) for professional use is that it is unstable when stored in a plastic container.Fluoride varnish contains a 2% sodium fluoride solution (NaF).

Both statements are FALSE.

Radiographically the carious lesion on tooth #2 appears to be larger than the carious lesion on tooth #30.The carious lesion on tooth number 20 appears to be in more immediate need of treatment than the carious lesion on tooth #14 (case F).

Both statements are FALSE. (case F)

Dental sealants are indicated in teeth with incipient pit and fissure carious lesions.The presence of caries-free pits and fissures with deep and irregular grooves is an indication for sealant application.

Both statements are TRUE

In dental radiography, interpretation refers to an explanation of what is viewed on a radiograph.The term diagnosis refers to the identification of a problem, deficiency/excess, or disease by examination or analysis.

Both statements are TRUE.

The prevalence and severity of periodontal diseases increases with both type 1 and type 2 diabetes.The presence of uncontrolled diabetes increases dental caries risk as a result of reduced saliva secretion and increased glucose in saliva

Both statements are TRUE.

The major cause for the error on the distal of the maxillary canine films is incorrect angulation of the x-ray beam. As seen in the intra oral photographs, there is not crowding of the anterior teeth (case E)

Both statements are TRUE. (case E)

When treating the maxillary arch, the patient should tilt the chin downward. When treating the mandibular arch, the patient should tilt the chin upward.

Both statements are false.

Glutaraldehyde is considered an intermediate-level disinfectant. Intermediate-level disinfectants can inactive bacterial spores.

Both statements are false. Glutaraldehydes are high-level disinfectants and can not kill bacterial spores.

The dependable variable is the outcome of interest. The independent variable is the intervention of the experiment.

Both statements are true.

Ultrasonic scalers are best to remove amalgam overhangs. Ultrasonic scalers should be avoided with veneers.

Both statements are true.

Alginate impressions must be removed with a quick snap because this minimizes distortion.

Both the statement and reason are correct and related

Open communication between a dental hygienist and patient minimizes misunderstandings and reduces the likelihood of litigation because a patient who senses professionalism and expertise may not be as prone to file a lawsuit in the event of an unsuccessful procedure.

Both the statement and reason are correct and related.

The anterior occlusal relationship between this patient's maxillary and mandibular anterior teeth is BEST described as normal overbite because the incisal edges of the maxillary teeth lie within the incisal third of the mandibular teeth. (case E)

Both the statement and reason are correct and related. (case E)

The fluticasone/salmeterol inhaler is prescribed for this patient to enhance which one of the following effects? (case J)

Bronchodilation (case J)

Which is the correct sequence of stages of tooth development

Bud, cap, bell, dentinogenesis, amelogenesis, appositional dentin and enamel, eruption and root development, functional stage

What is the Mohs hardness value for enamel? A. 2 B. 3 to 4 C. 5 to 6 D. 7 E. 9

C. 5 to 6

Which of the following would be the best indicator of treatment success at the revaluation appointment 4 to 8 weeks after initial periodontal therapy? A. Relatively free of plaque biofilm B. Stable periodontal probing depths C. Absence of gingival inflammation and bleeding D. No re-formed dental calculus deposits

C. Absence of gingival inflammation and bleeding

A dental hygienist is creating an oral health care program for participants at a local women, infants, and children program (WIC). The hygienist will develop a community profile to determine the participant's dental knowledge. instruction will include tooth brushing, oral physiotherapy techniques, and the relationship between diet and dental decay. The program will be offered for one month on days participants pick up food vouchers. by offering instruction at the WIC site, the dental hygienist hopes to educate caregivers and children early enough to prevent dental decay. Which provides information to parents on child developmental stages so the child's oral health needs can be adequately met? A. Defensive dentistry B. Preventive direction C. Anticipatory guidance D. Supervisory management

C. Anticipatory guidance

A fall in blood pressure could occur when a patient is experiencing each of the following conditions except one. Which one is the exception? A. Shock B. Syncope C. Anxiety D. Allergic reaction E. Addison's disease

C. Anxiety

Which toothbrushing method is most effective for cleaning around fixed orthodontic appliances? A. Bass B. Fones C. Charter D. Stillman

C. Charter

Both horizontal and vertical tooth mobility A. Class I B. Class II C. Class III D. None

C. Class III

Which instrument is the most effective to remove calculus in 6 mm pockets on the mesial of the distobuccal root of tooth #14? A. Gracey 5/6 B. Gracey 17/18 C. Gracey 15/16 D. Gracey 11/12

C. Gracey 15/16

Which classification does the American society of anesthesiologists (ASA) assign to a patient with controlled angina pectoris? A. I B. II C. III D. IV

C. III

Which organ is involved in the "first-pass" effect after oral administration of a drug? A. Kidney B. Lungs C. Liver D. Spleen

C. Liver

You have been employed as a public health dental hygienist in a local health department to provide educational presentations for the participants in the WIC non-fluoridated program in a non-fluoridated community. The natural fluoride (f) concentration of the community water supply is 0.3ppm f. one of your first assignments is to present an educational program on basic oral health practices for culturally diverse pregnant teens in three alternative high schools that have daycare facilities. The most frequently occurring score in a distribution of your program's data is which of the following? A. Mean B. Median C. Mode D. Range E. Standard deviation

C. Mode

Fetid breath odor, pseudomembranous ulceration at the marginal gingival an interdental papillae, gingival bleeding, and the presence of spirochetes and fusiform bacteria are associated with: A. Periapical abscess B. Herpes simplex I C. Necrotic ulcerative gingivitis (NUG) D. Aphthous ulceration E. Pericoronitis

C. Necrotic ulcerative gingivitis (NUG)

A 53-year-old client presents for a periodontal maintenance appointment. When developing a dental hygiene care plan, which of the following should be addressed first? A. Chlorhexidine stain on teeth B. Localized light plaque C. Pain on tooth #28 D. Smoking habit

C. Pain on tooth #28

From which approach is the mesial furcation of tooth #3 examined? A. Distal B. Mesial C. Palatal D. Buccal

C. Palatal

Each of the following usually results from parafunctional occlusal habits except A. Wear facets B. Mobile teeth C. Periodontal pockets D. Temporomandibular disorder

C. Periodontal pockets

Which phase of dental care focuses on periodontal surgery? A. Preliminary phase B. Phase I C. Phase II D. Phase III

C. Phase II

In leukoedema, the generalized opalescent condition caused by significant intracellular edema occurs in which layer of the stratified epithelium? A. Corneum B. Basal C. Prickle D. Granular

C. Prickle

The use of a scavenging system with N2O sedation equipment is desirable because it A. Removes the odor of nitrous from the room B. Assists the patient's respiration C. Removes any potentially toxic waste gas from the environment D. Allows the patient to inhale room air through the nosepiece.

C. Removes any potentially toxic waste gas from the environment


Related study sets

Corporate Finance for Managers Chapter 15 Study Guide

View Set

Chapter 29: Trauma Systems and Mechanism of Injury

View Set

Ordering, Comparing and Changing Mixed Fractions

View Set

PROPHETS: ZEPHANIAH, HABAKKUK,M JEREMIAH

View Set

HESI Patient Review: Terry Johnson

View Set